Immunology MCQ

Pataasin ang iyong marka sa homework at exams ngayon gamit ang Quizwiz!

*The answer B.* Numbers of circulating neutrophils (mostly segmented form) quickly increase on an acute infection. Such numbers are recruited that some juvenile forms are pressed into the circulation prior to their maturation. B and T lymphocytes, natural killer (NK) cells, and monocytes and macrophages do not show the same rate of increase.

Three days ago, an otherwise healthy 1 7-year-old boy sustained a skin laceration d u ring a lacrosse match. Yesterday, he complained of mild "flu like" symptoms. This morning, he became suddenly ill with a fever, general muscle aches, and dizziness; then he lost consciousness. On arrival in the emergency department, he had a temperature of 37.8°C and a heart rate of 136 beats per minute. His blood leukocyte count was 22,000 cells per f1l (reference range: 4,500 to 1 2,500 per f1l). The predominant cell type(s) in this patient's blood is/are most likely A. B lymphocytes. B. juvenile and mature neutrophils. C. monocytes and macrophages. D. natural killer cells. E . T lymphocytes.

*The answer is E.* IL-17 promotes inflammatory action such as the development of M2 macrophage and recruitment of eosinophils.

A pro-inflammatory cytokine with a major role in asthma is a (A) INF-γ (B) IL-4 (C) IL-6 (D) IL-10 (E) IL-17

*The answer is D.* B7 (choice A), expressed on antigen presenting cells, provides costimulatory signals for T-cell activation/suppression depending upon the interacting receptor. CD40 (choice B) is the receptor for CD40L. Signals generated by CD40L-CD40 interactions result in activation as in the case for B cells or maturation when occurring in dendritic cells. CTLA-4 (choice C) signaling medi-ated by binding with B7 from antigen presenting cells, suppresses T cell activation. FcR (choice E) are surface receptors that bind to Fc regions of antibodies and result in antibody-dependent cell cytotoxicity (ADCC) or opsonization. Only Fas-FasL interactions result in apoptosis caspase activation.

The interaction of which molecule on the membrane of cells with its ligand signals apoptosis? (A) B7 (CD80/86) (B) CD40 (C) CTLA-4 (CD152) (D) Fas (CD95) (E) Fc receptor (CD16)

*The answer is B.* Positive and negative selection processes make up the event termed central tolerance whose goal is to remove self-antigen-specific T cells. Allergens (choice A) are environmental antigens which are recognized by IgE molecules bound to Fc receptors on mast cells. Haptens (choice C) are chemicals which bind to carrier proteins and can then be immunogenic. Immunogens (choice D) are those antigens which promote immunity. The term antigens (choice E) refers to all chemical structures which can elicit immune responses, whether self- or foreign-derived in nature. Only tolerogens refers to self-antigens which are presented by thymic antigen presenting cells in order to facilitate negative selection.

Thymocytes interacting with self-peptides undergo negative selection. The self-peptides in this reaction act as: (A) Allergens (B) Tolerogens (C) Haptens (D) Immunogens (E) Antigen

*The answer is C.* C5a is a chemotactic agent and an anaphylatoxin.

During an experiment, an investigator gently abrades the skin from the flank of a mouse, creating a 1 × 2-cm skin window. A glass coverslip is then placed over the area so that cells attracted to the site attach to the coverslip for assessment. Two hours later, an extravasation of cells from the vasculature is noted on the coverslip. Which of the following complement components is the direct cause of the enhanced vascular permeability and chemoattraction in the abraded skin area in this experiment? (A) C1q (B) C4b (C) C5a (D) C7 (E) C9

*The answer is B.* MCTD has features of other common autoimmune diseases (e.g., SLE and scleroderma) but appears to be distinct. Patients typically have autoantibodies to ribonucleoproteins, but unlike SLE, they do not have antibodies to Sm antigen or double-stranded DNA. Some patients with MCTD develop symptoms of scleroderma or rheumatoid arthritis, suggesting that MCTD may be an intermediate stage in a genetically determined progression. Whether MCTD represents a distinct entity or simply an overlap of symptoms in patients with other types of collagen vascular diseases remains an open question. Intermittent episodes of ischemia of the fingers, marked by pallor, paresthesias, and pain, are referred to as Raynaud phenomenon. None of the other choices feature this constellation of signs and symptoms. Diagnosis: Mixed connective tissue disease

A 50-year-old woman complains of intermittent tingling and pain in the tips of her fingers. She also reports joint and muscle pain. Physical examination reveals lymphadenopathy. Laboratory studies show hypergammaglobulinemia. The antinuclear antibody test is positive, but there is no evidence of antibodies against double-stranded DNA. Urinalysis is normal. The patient responds well to steroids. Which of the following is the most likely diagnosis? (A) Graves disease (B) Mixed connective tissue disease (C) Myasthenia gravis (D) Scleroderma (E) Sjögren syndrome

*The answer is D.* SCID is a group of disorders of T and B lymphocytes that are characterized by recurrent viral, bacterial, fungal, and protozoal infections. Many infants with SCID have severely reduced volumes of lymphoid tissue and an immature thymus that lacks lymphocytes. In some patients, lymphocytes fail to develop beyond pre-B cells and pre-T cells. About one half of these severely immunodeficient children lack adenosine deaminase (ADA). ADA deficiency causes the accumulation of intermediate products that are toxic to lymphocytes. These children cannot survive beyond early infancy unless they are raised in a sterile environment ("bubble children"). None of the other choices are associated with ADA deficiency. Diagnosis: Severe combined immunodeficiency

A 9-month-old girl with a history of recurrent pulmonary infections is found to have a congenital deficiency of adenosine deaminase, which is associated with a virtual absence of lymphocytes in her peripheral lymphoid organs. What is the appropriate diagnosis? (A) Bruton X-linked agammaglobulinemia (B) DiGeorge syndrome (C) Isolated IgA defi ciency (D) Severe combined immunodefi ciency (E) Wiskott-Aldrich syndrome

*The answer is B.* Immature dendritic cells at the infection site are triggered to mature and migrate to the lymph node. Immature dendritic cells have low expression of MHC Class II, but mature cells have high expression levels of both MHC Class II and CD80/88.

A cell expressing cell surface MHC Class II, CD80/88 and secretes IL-12 is a(n) (A) Immature myeloid-derived dendritic cell (B) Mature myeloid-derived dendritic cell (C) Follicular dendritic cell (D) B cell (E) Macrophage

*The answer is E.* Plasma cells are terminal and dedicated to producing immunoglobulin; no surface receptor is needed to direct further development.

A cell that secretes large quantities of antibody but does not express surface immunoglobulin is a (A) B-1 cell (B) Naïve mature B cell (C) Centrocyte (D) Memory B cell (E) Plasma cell

*The answer is A.* INF-γ regulates cell-mediated defenses.

A cytokine that promotes cell-mediated immunity and is produced by Th1 cells is a (A) INF-γ (B) IL-4 (C) IL-6 (D) IL-10 (E) IL-17

*The answer is E.* Complement proteins function in innate immunity through enzymatic cleavage of C3 to liberate C3b which directly binds to pathogen surfaces. Following C3b insertion, enzyme cascade events result in lysis of the target cell by the membrane attack complex (complement protein scaffold C5-C9) and recruitment of phagocytes by C3a and C5a cleavage products. Chronic eczema (choice A), immune hemolytic anemia (choice B), incomplete recovery from viral infections (choice C), and poor response to vaccination (choice D) would not involve complement pathways and thus would be unaffected by a C3 deficiency.

A workup on an ill child revealed low levels of complement C3 in her blood. Which one of the following presentations did this child most likely manifest? (A) Chronic eczema (B) Immune hemolytic anemia (C) Incomplete recovery from viral infections (D) Poor response to vaccination (E) Recurrent infections with extracellular bacteria

*The answer is B.* Recurrent viral and fungal infections suggest a defect in the T-cell compartment. This suggestion is confirmed by lab results indicating normal levels of B cells but reduced numbers of T cells. Radiograph reveals that an abnormal development of thymic tissue and suggest a diagnosis of DiGeorge syndrome. Since the patient has had no severe complications from past infections and responds well but slowly with pharmacological intervention, continuing this treatment strategy should be sufficient therapy as T-cell function often improves by the age of 5 years in these patients. Acquisition of such T-cell improvement is probably due to as yet unidentified maturation site for the T cells. If past infections had been severe to life-threatening, bone marrow or fetal thymic transplantation would be an appropriate mode of action. Intravenous IL-2 will be ineffective in expanding T-cell numbers as the T cells in the patient are immature and do not express the receptor to respond to this cytokine. In addition, the small percentage of T cells that express the IL-2 receptor constitutively are regulatory cells and activation with this cytokine may dampen any response to infectious challenge resulting in more severe disease. Due to the lack of T cells required for clearance of viral and fungal challenges, no treatment would result in pro-longed and perhaps increased severity of all future infections.

An 18-month-old male child has a history of recurrent viral and fungal infections including respiratory syncytial virus and Candida. Examination revealed a marked reduction in T lymphocyte absolute numbers but a normal level of B cells. Furthermore, a chest radiograph reveals that the patient's thymus is hypoplastic. Assuming past infections were not severe, which of the following is the most appropriate therapeutic modality? (A) No treatment (B) Pharmacological intervention for future infections (C) Bone marrow transplant (D) Fetal thymic transplantation (E) Intravenous interleukin-2 (IL-2)

*The answer is B.* Type III hypersensitivity reactions are characterized by immune complex deposition, complement fixation, and localized inflammation. Antibody directed against either a circulating antigen or an antigen that is deposited in a tissue can give rise to a type III response. Diseases that seem to be most clearly attributable to the deposition of immune complexes are systemic lupus erythematosus, rheumatoid arthritis, and varieties of glomerulonephritis. Streptoccocal infection in this case led to the deposition of antigens and antibodies in glomerular basement membranes, resulting in clinical features of nephritic syndrome (e.g., hematuria, oliguria, and hypertension). Poststreptococcal illnesses do not include any of the other choices. Diagnosis: Postinfectious glomerulonephritis

An 8-year-old boy presents with periorbital edema and throbbing headaches. His parents report that the boy had a "strep throat" 2 weeks ago. Urinalysis shows 3+ hematuria. A renal biopsy shows hypercellular glomeruli, and electron microscopic examination of glomeruli discloses subepithelial "humps." Which of the following best explains the pathogenesis of glomerulonephritis in this patient? (A) Antineutrophil cytoplasmic autoantibodies (B) Deposition of circulating immune complexes (C) Directly cytotoxic IgG and IgM antibodies (D) IgE-mediated mast cell degranulation (E) T cell-mediated delayed hypersensitivity reaction

*The answer is D.* The epitope specific receptors of T cells (TCRs) are displayed as membrane-bound molecules on their cell surfaces. TCRs are not found as soluble molecules. Epitope-specific molecules produced by plasma cells are genetically distinct from T cell receptor molecules.

Epitope-specific receptors of T lymphocytes are found A. as either cytosolic or membrane-bound proteins. B. in blood plasma, lymph, and other secretory fluids. C. on the surface of plasma cells. D. as transmembrane polypeptides. E . in the nuclear lipid bilayey

*The answer is A.* CD4 (choice B) functions to stabilize interactions of TCR and MHC during T-cell activation. CD28 (choice C) is the costimulatory receptor for B7 molecules which promotes activation of T-cell responses. CD45 (choice D) is a cell surface marker of activation status but does not function in TCR signaling. CD152 or CTLA-4 (choice E) is a cos-timulatory receptor for B7 molecules which promotes suppression of T-cell responses.

The intracellular signal initiated by antigen binding to the T-cell receptor is generated by which set of molecules expressed on the T cell membrane? (A) CD3 (B) CD4 (C) CD28 (D) CD45 (E) CD152

*The answer is D.* This transcription factor, along with AP-1 and NFAT (nuclear factor of activated T cells), promotes activation of genes for pro-inflammatory cytokines and proteins necessary for cell activation and proliferation. They are inhibited by corticosteroids. FoxP3 is a transcription factor found in regulatory T cells. The other molecules listed are not transcription factors, but are instead involved with signal transduction following antigen binding to T-cell receptors.

The repression of which transcription factor explains on a molecular level the anti-inflammatory effect of corticosteroids? (A) FoxP3 (B) ITAM (C) MAP kinase (D) NF-κB (E) Protein kinase C

*The answer is E.* B cells (choice A) can produce IL-4, IL-5, and IL-10. Macrophages (choice B) can produce IL-4 and IL-10. Mast cells (choice C) can produce IL-4. Th1 (choice D) cells do not produce any of these cytokines as these factors tend to sup-press Th1 activation and differentiation. Interestingly, natural killer T cells can produce these cytokines as well upon initial activation in the presence of IL-4 production by macrophages, B cells, or mast cells.

Which cells are the source of interleukin-4, −5, −10, and −13? (A) B cells (B) Macrophages (C) Mast cells (D) Th1 cells (E) Th2 cells

*The answer is A.* CD4+ T cells are also called T-helper cells. Cos+ T cells have cytotoxic or suppressive functions. GlyCAM-1 is an adhesion molecule found on certain vascular epithelial cells within lymph nodes. lgA and lgG are not expressed on T cells.

Which of the following molecules is expressed by a mature T cell that functions as a helper T cell? A. CD4 B. CDS C. GlyCAM-1 D. lgA E . lgG

*The answer is B.* TNF-α, IL-1, and IL-12 are produced by activated macrophages to promote Th1 differentiation and inflammation. B cells (choice A) do not produce TNF-α or IL-12 but can produce IL-1. Mast cells (choice C) can pro-duce only TNF-α and IL-1. Th1 (choice D) cells can produce TNF-α and IL-1 but not IL-12. However, activated CD8+ T cells (Tc1) can produce IL-12. Th2 (choice E) cells can only produce IL-1 of these three cytokines.

Which one of the following cells is the major source of tumor necrosis factor alpha (TNF-α), interleukin-1, and interleukin-12? (A) B cells (B) Macrophages (C) Mast cells (D) Th1 cells (E) Th2 cells

*The answer is B.* Granulocytes are immune cells that contain granules in their cytoplasms. These granules are filled with enzymes and other inflammatory mediators which are released upon activation of the granulocytes. Neutrophils, eosinophils and basophils are considered granulocytes. Although both NK cells and NKT cells have granules, they are referred to as large, granular lymphocytes, not granulocytes. Monocytic cells such as dendritic cells and macrophages (choices A and C) lack distinct cytoplasmic granules.

Which one of the following leukocytes is considered a "granulocyte"? (A) Macrophage (B) Neutrophil (C) Dendritic cell (D) Natural killer cell (E) Natural killer T cell

*The answer is D.* Tumor immunity requires inflammatory responses, namely, Th1 cells and IFNg-producing CTL (choices A and B). Tumor evasion tactics function to eliminate this immune response. In order to maximally activate those CTL, a specialized antigen presenting cell, the dendritic cell, can internalize exogenous tumor antigens, reroute those antigens to the class I MHC pathway, and present effectively to CD8+ T cells to generate tumor-specific CTL (choice C). This process of rerouting exogenous antigens to the class I MHC pathway is called cross-presentation and the dendritic cell is the only cell capable of this function. In order for a tumor to grow in size, it requires a large supply of nutrients obtained by the construction of new blood vessels to service the tumor directly (angiogenesis) (choice E). In addition, these new blood vessels can limit the accessibility of tumor-specific cells into the tumor. A number of antiangiogenesis drugs are in clinical trials to block this mechanism of tumor escape. One of the primary mechanisms of tumor escape from CTL responses is to downregulate gene or surface expression of class I MHC. This tends to render the tumors susceptible to lysis by NK cells; however, the recruitment of Th2 and regulatory T cells (by tumor products) to the tumor site has been implicated to impede NK cell activities.

Which one of the following represents a mechanism of tumor evasion from the immune response? (A) Deviation of helper T cells to type-1 or Th1 cells (B) Production of interferon-γ (IFN γ) by the tumor (C) Cross-presentation of tumor antigen by dendritic cells to activate CD8+ T cells (D) Decreased expression of class I MHC on tumor cells (E) Tumor inhibition of angiogenesis

*The answer is A.* The initial recruitment of neutrophils to an inflammatory site requires CCL8 release by resident macrophage or mast cells to initiate cellular extravasation.

A short-lived phagocytic cell recruited to inflammatory sites by macrophage secretion of CCL8 is a (A) Neutrophil (B) Basophil (C) Eosinophil (D) Plasmacytoid cell (E) Mast cell

*The answer is C.* Thymus-derived or T cells are a subset of lymphocytes. None of the other cell types differentiate within the thymus.

A subset of which of the following of these undergoes further differentiation within the thymus? A. Basophils B. Eosinophils C. Lymphocytes D. Monocytes E. Neutrophils

*The answer is C.* MHC-II on macrophages presents processed peptides to T cells.

An investigator injects an experimental animal with a newly discovered bacterial strain to evaluate T-lymphocyte activation. It is found that bacterial engulfment by macrophages results in the presentation of bacterial-derived peptide ligands to CD4+ T lymphocytes. Which of the following cell-surface molecules on the macrophage is most directly involved in the presentation of the processed peptides? (A) CD28 (B) Class II MHC (C) Fcε receptor (D) Interleukin-2 (IL-2) receptor (E) Membrane immunoglobulin

*The answer is D.* With regard to the pathogenesis of septic shock, TSST-1 acts as a polyclonal activator, stimulating a number of different T cells in a non-specific fashion and resulting in cytokine secretion. The hypersecretion of pro-inflammatory cytokines such as tumor necrosis factor-alpha can lead to septic shock as occurred in this patient. Anergic cells would not participate in the pathogenesis of this condition. The other choices are not typical responses of T cells to superantigen stimulation and would also not play a role in the pathogenesis of septic shock induced by TSST-1.

What response of T cells plays a role in the pathogenesis of shock following the binding of TSST-1 and similar molecules? (A) Anergy (B) Apoptosis (C) Cell cytotoxicity (D) Cytokine secretion (E) Memory cell differentiation

*The answer is B.* IL-4 regulates humoral defenses.

A cytokine that promotes humoral immunity and is produced by Th2 cells is a (A) INF-γ (B) IL-4 (C) IL-6 (D) IL-10 (E) IL-17

*The answer is B.*T umors have numerous mechanisms to escape immune responses. The most important antitumor immune response is the activation of tumor-specific CD8+ T cells. It is the CTL which eradicates the tumor from the patient and the focus of much cellular immunotherapies (infusion of modified CTL into tumor patients). Since CTL are responsible for eradication of tumors and these cells require antigen recognition for targeting their cytotoxic machinery, these cells would need to interact with tumors expressing class I MHC (choices A, C, and D). Only tumors expressing low levels of class I MHC are capable of escaping CTL-mediated killing. Even moderately class I MHC-expressing tumors are susceptible to CTL lysis.

Tumor editing is one of the most difficult clinically relevant problems to overcome in the course of tumor therapy. Which one of the following accounts for the mechanism of tumor editing? (A) Survival of tumors with moderate expression of class II MHC molecules (B) Survival of tumors with low expression of class I MHC molecules (C) Survival of tumors with low expression of class II MHC molecules (D) Survival of tumors with high expression of class II MHC molecules (E) Survival of tumors with moderate expression of class I MHC molecules

*The answer is E.* Lymphocytes traffic between the blood and lymph circulation in order to maintain surveillance of the body for foreign invaders. Lymphoid and germinal center follicles (choices A-C) are areas in the lymph node where B cells and plasma cells, respectively, are found in high concentration. Periarterial lymphatic sheath (choice D) is the site in the spleen white pulp where T cells are heavily concentrated. In order for T cells to gain access to the lymph nodes from the circulation, specialized zones called high endothelial venules provide connections from the blood to the lymph node. The cells must arrest at the HEV via integrin binding, through CD62L found on naïve T cells, and then those arrested cells follow a chemokine (chemotactic) gradient extravasating through the endothelial cells of the HEV into the cortex of the lymph node.

Which of the following is the site at which lymphocytes can leave the blood and gain entry into the lymph nodes and what lymphocyte cell surface protein mediates such access? (A) Lymphoid follicle: CD4 (B) Germinal center: CD62L (L-selectin) (C) Lymphoid follicle: CD62L (L-selectin) (D) Periarterial lymphatic sheath (PALS): CCR7 (E) High endothelial venules (HEV): CD62L (L-selectin)

*The answer is C.* Mature T cells (both CD4+ and CD8+) express CD3, a molecular complex associated with the TCR. CD4+ cells are T cells with T helper function and do not express B-cell receptors. CD1d is a specialized, nonclassical MHC class I molecule on NKT cells. CD8 is a molecule expressed by T cytotoxic and suppressor cells. CD19 is expressed on B cells.

Which of the following molecules is expressed on the surface of mature CD4 + cells? A. B-cell receptor B. CD1d C. CD3 D. CD8 E. CD19

*The answer is B.* The committing step for helper T cell activation is the simultaneous expression of the high affinity IL-2 receptor subunit CD25 and the release of IL-2 to provide autocrine stimulation of proliferation

A CD3+ cell that secretes IL-2 when activated is a (A) (γδ) T cell (B) Helper T cell (C) Cytotoxic T cell (D) Regulatory T cell (E) Natural killer T cell

*The answer is A.* A characteristic of γδ T cells is to surface on the mucosal barrier of the gut.

A T cell located at the epithelial barrier of the gut is a (A) (γδ) T cell (B) Helper T cell (C) Cytotoxic T cell (D) Regulatory T cell (E) Natural killer T cell

*The answer is C.* Activation of a naive B cell requires both the engagement of its B cell receptor (immunoglobulin) and the receipt of secondary signals from CD4+ Th2 cells. The B cell does not require interaction with antigen-presenting cells such as dendritic cells. The lgD and lgM on its surface have the same epitope specificity. Turn over of cytoplasmic molecules by proteasomes is a normal on going activity but is not involved in the naive B cell's activation. The isotype switch occurs only during the reactivation of memory B cells, not during the initial activation of naive B cells.

Activation of an individual naive B cell involves binding of membrane-associated epitopes leading to A. dendritic cell presentation of MHC class I. B. recognition of different epitopes by surface lgD and lgM. C. signaling from both the B-cell receptor and a CD4 + Th2 cell . D. the isotype switch. E. ubiquitination and destruction of antigen by proteasomes.

*The answer is D.* IL-10 is produced by regulatory T cells to suppress immunity.

An anti-inflammatory cytokine is a (A) INF-γ (B) IL-4 (C) IL-6 (D) IL-10 (E) IL-17

*The answer is D.* B cells synthesize and express immunoglobulin on their cell surfaces. Immunoglobulins with in an individual B cell contain specificity for one epitope, not several. Cytoplasmic phagosomes are involved in degradation of unwanted materials. Membrane complexes also containing CD3 are T-cell receptors (TCR) on the surfaces of T cells. B cells express surface lgM before leaving the bone marrow.

B lymphocytes synthesize and express immunoglobulin A. containing multiple epitope specificities. B. in cytoplasmic phagosomes. C. in membrane complexes also containing CD3. D. on their cell membrane surface. E. only after leaving the bone marrow.

*The answer is D.* STAT dimers translocate into the nucleus. JAKs are cytosolic tyrosine kinases that bind to the intracellular domain of the tyrosine-phosphorylated receptor and never enter the nucleus. Ras is a membrane-bound GTP binding protein that is bound by cytosolic proteins with SH2 domains that also bind to phosphotyrosine residues within the intracellular portion of the receptor. Catalytic receptors signal by stimulating tyrosine kinase, either of the receptor itself (intrinsic activity) or by associating with nonreceptor tyrosine kinases (e.g., JAK), neither of which enters the nucleus.

Following cytokine binding to a specific cell-surface receptor, a lymphocyte is stimulated to undergo signaling via the JAK-STAT pathway. In this pathway, which of the following will be induced to translocate to the cell's nucleus to regulate transcription? A. JAK B. Ras C. SH2-containing adapter proteins D. STAT dimers E. tyrosine kinase

*The answer is C.* Cytoplasmically derived peptides are presented by MHC I molecules. Pattern recognition receptors do not present peptides to T cells nor do γδ T cells. CD8+ T cells recognize peptide fragments presented by class I MHC molecules. They are not processed in endocytic vesicles for presentation by MHC II molecules to CD4+ T cells.

Fragments of a cytoplasmic pathogen are presented to T cells by A. direct engagement of cell surface pattern recognition receptors. B. macropinocytosis into γδ T cells. C. MHC class I molecules to CD8+ T cells. D. phagocytosis and presentation to CD4 + T cells. E. placement into endocytic vesicles and complexing with MHC class II molecules.

*The answer is E.* Following activation of a T cell , CD152 begins to move from the Golgi apparatus out onto the cell surface, where it competes with CD28 for binding of CD8O/ CD86 on antigen-presenting cells. It does not remain sequestered in the Golgi, nor does it bind to pMHC. Its binding induces an inhibition of IL-2 mARA and the progression of the T cell through the cell cycle.

In activated T cells, CD152 (CTLA4) A. becomes sequestered within the Golgi. B. binds to the appropriate surface pMHC. C. induces progression through the cell cycle. D. stimulates transcription of IL-2 mRNA. E. begins to move to the membrane and bind CD80/86.

*The answer is A.* IL-6 induces production of C-reactive protein by the liver. It does not induce the liver to produce chemokines, complement, immunoglobulins, or interleukins.

In response to the lipopolysaccharide from a gram-negative bacterial infection, local host phagocytes release proinflammatory cytokines, including IL-6, which then stimulates hepatic synthesis and release of A. C-reactive protein. B. chemokines. C. complement. D. immunoglobulins. E. interleukins.

*The answer is A.* Lymph nodes are divided into the cortex and the medulla. Lymph is the watery fluid of the lymphatic circulatory system that contains leukocytes and cellular debris from various organs and tissues. Reticulum refers to the framework of a lymph node that is composed of phagocytes and specialized kinds of reticular or dendritic cells.

Lymph nodes have two main regions: the A. cortex and medulla. B. lymph and cortex. C. reticulum and cortex. D. lymph and medulla. E. reticulum and medulla.

*The answer is E.* Selectins are adhesion molecules that participate in the recognition that occurs between different types of cells and tissues. Antibodies do not serve as guides for such homing. CD8 and CD3 are expressed on T cells, not on B cells, and are responsible for lymphocyte homing. Complement fragments may be chemoattractants for leukocytes, but they attract those cells to the site of immune responses rather than to specific organs.

Migration of a B lymphocyte to specific sites (such as a lymph node) is dependent in part on the use of A. antibodies. B. CDS. C. CD3. D. complement. E. selectins.

*The answer is C.* Natural killer (NK) cells are lymphocytes and are often described as large, granular, non-T, non-B lymphocytes. Although they do contain cytoplasmic granules, these are considerably less prominent than those found in granulocytes (basophils, eosinophils, and neutrophils) . Nor are NK cells members of the monocyte family.

Natural killer cells are members of which of the following families of leukocytes? A. Basophils B. Eosinophils C. Lymphocytes D. Monocytes E. Neutrophils

*The answer is E.* Pattern recognition receptors (PARs) are genomically determined to bind to molecules widely expressed by microbes but not by host cells. Consequently, PARs can not recognize host-associated molecules such as MHC class I molecules or cells of host origin such as B, T, or natural killer lymphocytes.

Pattern recognition receptors bind to A. B and T lymphocytes. B. host cell-associated molecules. C. MHC I molecules. D. natural killer cells. E. pathogen-associated molecular patterns.

*The answer is C.* IL-1α and β (choice A) are inflammatory cytokines produced by phagocytes (and others) to promote inflammation. IL-4 (choice B) supports differentiation of Th2-type CD4+ T cells, while IL-12 and IL-18 (choices D and E) operate to differentiate Th1-type CD4+ T cells.

Which of the following cytokines supports proliferation and differentiation of developing lymphocytes in the primary lymphoid tissue? (A) Interleukin-1 (B) Interleukin-4 (C) Interleukin-7 (D) Interleukin-12 (E) Interleukin-18

*The answer is E.* A marked increase in blood neutrophils is a hallmark of infection. Basophils and eosinophils are rarely seen in the circulation in numbers that exceed 5% of the bloodborne leukocytes. Monocytes and lymphocytes increase in notable numbers usually only in chronic disorders.

A 1 6-year-old boy has acute appendicitis (infection of the appendix). Which of the following blood cells is most likely to increase in number as a result of his condition? A. Basophils B. Eosinophils C. Lymphocytes D. Monocytes E. Neutrophils

*The answer is D.* Vaccinations are used to establish immunological memory to a particular pathogen.

A product of vaccination is (A) B-1 cells (B) Naïve mature B cells (C) Centrocytes (D) Memory B cells (E) Plasma cells

*The answer is B.* A tissue resident cell that responds to PAMPs and releases histamine and eicosanoids is a basophil.

A tissue resident cell that responds to PAMPs and releases histamine and eicosanoids is a (A) Neutrophil (B) Basophil (C) Eosinophil (D) Plasmacytoid cell (E) Mast cell

*The answer is D.* CD4+ Th1 cells recruit and activate macrophages to destroy intracellular pathogens. Antigen-presenting cells are not T cells. Cytotoxic T lymphocytes are CD8+. Th0 and Th2 cells, although also being CD4+ , do not engage in this activity.

CD4 + T cells that respond to intracellular pathogens by recruiting and activating phagocytic cells are termed A. antigen-presenting cells. B. cytotoxic T lymphocytes. C. Th0 cells. D. Th1 cells. E. Th2 cells.

*The answer is E.* The defective thymic environment inhibits T cell development and function. Because so much B cell activity depends on interaction with T cells, B cell responses will also be impaired. Complement would not be impaired while sparing T and B cell activity. NK cell function should not be affected.

DiGeorge syndrome is an immune deficiency disease resulting from impaired thymic development. Which of the following is/are affected in patients with DiGeorge syndrome? A. B cell development only B. complement only C. NK cell function D. T cell development only E. T cell and B cell functions

*The answer is B.* A hallmark of the adaptive immune system is that it progressively alters its response upon reexposure to an antigenic stimulus, and in doing so, it must recall the previous exposure, a process known as memory. Although they are members of the innate immune system and do not possess immunologic memory, phagocytes may be influenced by the adaptive immune system. Consistency in immune response from initial to subsequent encounters is a hall mark of the innate immune response. Immunologic memory of the adaptive immune system is not passed genetically from one generation of individuals to the next. Detection of diminished MHC class I expression is a function of natural killer cells, members of the innate immune system .

Immunologic memory refers to A. activation of phagocytic cells to ingest microbial invaders. B. changes in adaptive immune responses with subsequent encounters with antigen. C. constancy of the response of the innate immune response to a particular microbe. D. recognition of pathogen-associated molecular patterns by pattern recognition receptors. E. stimulating a defective host cell with reduced MHC I molecules to commit suicide.

*The answer is C.* Cross-linking of lgE bound to the surfaces of basophils and mast cells causes cellular degranulation and release of vasoactive amine responsible for inflammation . In humans, neither lgA nor lgG is associated with allergic responses. Histamine is released from mast cells are a result of cross-linking of surface-bound lgE.

In an individual with an immediate hypersensitivity response (allergy) to dust mites, cross-linking of which of the following dust-mite-specific molecules will trigger inflammatory mediator release? A. histamine B. IgA C. IgE D. IgG E. mast cells

*The answer is B.* B cells, dendritic cells, monocytes, and macrophages constitutively express MHC class II molecules. Only a subset of nucleated cells expresses MHC class II molecules, and it does not include mast cells or naive T cells. Erythrocytes do not express MHC class II molecules.

In humans, MHC class II molecules are expressed by A. all nucleated cells. B. B cells, dendritic cells, and macrophages. C. erythrocytes. D. mast cells. E. naive T cells.

*The answer is A.* MHC class I molecules are self-identification molecules found on all nucleated host cells. Natural killer cells, after making contact with cells expressing stress signals, make the decision whether to kill them or not by assessing whether they express the appropriate types and levels of MHC I molecules. Although they are members of the innate immune system , natural killer cells do not recognize nonself, pathogen-associated molecular patterns, or pattern recognition receptors. Natural killer cells are unable to recognize somatically generated cell surface receptors.

Natural killer cells assess whether other cells are abnormal by detecting types and levels of surface-associated A. MHC class I molecules. B. nonself molecules. C. pathogen-associated molecular patterns. D. pattern recognition receptors. E. somatically generated cell surface receptors.

*The answer is E.* Negative selection of T cells occurs as they move from the thymic cortex into the thymic medulla. It does not occur at sites outside of the thymus.

Negative selection of T cells occurs in the A. blood vessels. B. bone marrow. C. lymph node. D. spleen. E. thymus.

*The answer is E.* Immature T cells (double-positive or CD4+CD8+) interact with MHC class I and II-positive thymic epithelial cells and following this interaction, the T cells assume single positive status (either CD4 or CD8 expression) and express a functional TCR. CD28 (choice A) is a T-cell surface molecule involved in costimulation signals; however, it is not relevant for positive selection. While Fc receptors (choice B) and MHC class I (choice C) molecules are expressed by T cells, Fc receptors serve as a means to interact with antibodies and MHC to present anti gens to T cells. There is, at present, some evidence that T cells can present antigens, albeit inefficiently, to other T cells, this interaction is included in the T-T interac-tions that have been described. T cells do not express MHC class II (choice D) molecules.

Positive selection in the thymus occurs when thymocytes express functional versions of which critical molecule? (A) CD28 (B) Fc receptor (C) MHC class I (D) MHC class II (E) T-cell receptor (TCR)

*The answer is C.* Pre-pro-B cells initially express lgα and lgβ molecules. The synthesis of heavy and light chains (including su rrogate light chains) occurs at later stages of development.

Pre-pro-B cells A. contain either K or A. light chains. B. demonstrate surface expression of pseudo-lgM. C. express lgα and lgβ BCR accessory molecules. D. have VDJ joining of genes. E. express surrogate light chains.

*The answer is E.* Bone marrowderived (B) and thymus-derived (T) lymphocytes somatically generate receptors during development. Unlike natural killer cells, B cells and T cells are unable to assess the quantity of MHC class I molecules on nucleated cells. Unlike innate immune system receptors, B- and T-lymphocyte somatic receptors are randomly generated and vary greatly between individuals. B- and T-lymphocytes receptors are formed prior to antigen stimulation.

Somatically generated receptors found on B and T lymphocytes are A. bound only to MHC I molecules. B. encoded in the germline to recognize pathogen-associated molecular patterns. C. first produced after an initial encounter with nonself. D. identical among individuals. E. randomly generated during development.

*The answer is B.* Complement functions to facilitate the lysis of microbes by recognition of microbes tagged by antibody, by the opsonization of microbes by the attachment of C3 fragments, and by the release of anaphylotoxins C3a, C5a, and C4a. Immunoglobulin molecules are not cleaved by complement. The classical pathway is activated only by antigen-antibody complexes and by itself does not recognize microbial epitopes. Complement is not involved in lymphocyte development and does not trigger the release of histamine.

The classical pathway of complement functions to A. cleave immunoglobulins into Fc fragments. B. facilitate destruction of microbes. C. recognize specific epitopes on microbes. D. regulate lymphocyte development. E. trigger histamine release.

*The answer is D.* Primary lymphoid organs are sites where lymphocytes undergo their initial differentiation. Adaptive immune responses are initiated by mature lymphocytes that have migrated out of primary lymphoid organs. Secondary lymphoid organs contain filtration devices to remove foreign materials. Circulating leukocytes are found within blood and lymph and secondary lymphoid organs but not within primary lymphoid organs. Pattern recognition receptors (PRRs) are expressed by cells of the innate immune system.

The primary lymphoid organs are those in which A. adaptive immune responses are usually initiated. B. filtration devices remove foreign matter. C. large numbers of circulating leukocytes make contact with one another. D. lymphocytes undergo their initial differentiation. E. pattern recognition receptors bind antigens

*The answer is D.* Opsonization is the increased phagocytic uptake of cells or molecules tagged by antibodies (usually lgG1 ) or membrane-bound C3b or C4b. Agglutination is the aggregation or clumping of cells or particles bound together by antibodies (usually lgM or dimeric lgA) . Complement activation is initiated by the attachment of the C1 component of complement to epitope-bound antibody (lgM or lgG). Neutralization is the blocking by antibody of structures on microbes and toxins that allow them to bind to host cell surfaces. The precipitin reaction results from the assembly of large antigen-antibody complexes that precipitate from solution.

The process that is synergistically enhanced by the binding of both antibodies and complement fragments such as C3b by phagocytes is known as A. agglutination. B. complement activation. C. neutralization . D. opsonization. E. precipitin reaction.

*The answer is E.* The thymus is the site of initial differentiation of T cells. Erythrocytes develop from erythroid precursors in the bone marrow. Hematopoietic stem cells differentiate along any one of several lineages within the bone marrow. Natural killer (NK) cells develop within the bone marrow and lack rearranged TCR.

The thymus is the site of initial differentiation for A. B cells. B. erythrocytes. C. hematopoietic stem cells. D. NK cells. E. T cells.

*The answer is D.* The white pulp of the spleen is enriched in plasma cells secreting immunoglobulin , in addition to B and T lymphocytes. Erythrocytes are found within the red pulp of the spleen. CD4+CD8+ T cells are found in the thymus. Natural killer cells function within peripheral blood. Precursors of B cells are located in the bone marrow.

The white pulp of the spleen is enriched i n A. erythrocytes carrying hemoglobin. B. CD4+CD8+ T cells binding to MHC. C. natural killer cells recognizing targets. D. plasma cells secreting immunoglobin. E . precursor cells developing into mature B cells.

*The answer is C.* Once activated, cytotoxic T lymphocytes can bind and destroy infected cells expressing pMHC I complexes recognized by their T-cell receptors. Neither B cells nor CD4+ T cells recognize pMHC I. T helper cells-whether ThO, Th1 , or Th2-are CD4+ and do not recognize pMHC I .

Upon encountering an appropriate pMHC I on an infected cell, A. B-cell receptors become cross-linked and signaling ensues. B. CD4+ cells release IL-4. C. CD8 + cytotoxic T cells destroy the infected cell . D . naive Th1 cells secrete cytokines. E. Th0 cells differentiate into Th2 cells.

*The answer A.* Failure to bind IL-7 dooms the developing thymocyte. It will be unable to develop into either an αβ or a -γδ thymocyte. This interaction occurs after migration of the thymocytes into the thymus. Thymocytes cannot switch to the B cell developmental pathway.

What will be the fate of an early thymocyte that fails to express IL-7 receptors? A. apoptotic cell death B. development as a -γδ T cell C. development as an NKT cell D. failure to traffic to the thymus E. maturation along the B-cell lineage

*The answer is A.* IL-3 (choice B) is a differentiation factor for plasmacytoid dendritic cells. IL-4 and IL-5 (choices C and D) are factors involved in the differentiation and growth, respectively, of Th2 CD4 T cells. IL-6 (choice E) is a proinflammatory cytokine that has been reported to suppress regulatory T cell activities and in conjunction with TGF-β can differentiate Th17 CD4+ T cells.

Which cytokine is essential for T-cell proliferation and is also necessary for the production of CD25-positive regulatory T cells? (A) IL-2 (B) IL-3 (C) IL-4 (D) IL-5 (E) IL-6

*The answer is E.* The classical pathway of complement is initiated by the interaction of C1 (followed by C4, C2, and C3) with epitope-bound lgG or lgM. lgA, lgD, and lgE do not bind to C1.

Which of the following antibody isotypes facilitate the sequential binding of the C1, C4, C2, and C3 components of the complement system? A. lgA and lgD B. lgA and lgE C. lgA and lgM D. lgE and lgG E. lgG and lgM

*The answer is D.* T cells are found predominantly in the respiratory organs, skin, and peritoneal cavity. Their recognition repertoire is far less extensive that found in αβ T cells. They do not express significant immunologic memory but do react to antigenic stimuli more rapidly than do αβ T cells.

γδ T cells A. contain very extensive antigen recognition repertoires. B. express surface markers that are also characteristic of NK cells. C. generate memory when recognizing antigen on multiple occasions. D. migrate preferentially to respiratory organs, skin, and peritoneal cavity. E. respond more slowly to antigen than do αβ T cells

*The answer is B.* CD4+CD25+ T regulatory (Treg) cells or natural Treg cells function in peripheral tolerance to prevent self-antigen-specific T-cell activation. This is accomplished by still undefined cell-contact inhibitory mechanisms. What has been determined is that those inhibitory mechanisms are a product of the Foxp3 gene; thus, cells with active gene expression of Foxp3 (as determined by quantitative PCR) are regulatory T cells. Foxp3 is a unique gene to Treg cells and not expressed in other cell types.

A 1-year-old female presented with symptoms of systemic autoimmunity (lupus, diabetes, and arthritis), and upon genetic analysis it was found that the patient had a mutation in the Foxp3 gene locus. Which one of the following is an explanation for the clinical symptoms observed? (A) Patient cannot produce antigen presenting cells (B) Patient cannot generate regulatory CD4+CD25+ T cells (C) Patient is unable to produce antibodies (D) Patient cannot produce anti-inflammatory cytokines (E) Patient NK cells have no inhibitory signal

*The answer is A.* Susceptibility to bacterial infections implicate B-cell defects (lack of tonsil and lymph node tissue), while increased viral and fungal infections characterize a defect in the T-cell compartment. All of the answers result in severe combined immunodeficiency; however, only adenosine deaminase deficiencies will present with marked decreases in both T- and B-cell numbers. Mutations in CD25, JAK 3, CD127, and TBX1 result in reduced T-cell numbers, while B-cell counts remain in the normal range or may be slightly increased. In all cases, serum immunoglobulin levels are reduced.

A 1-year-old male born to term apparently healthy has for the last 5 months suffered from numerous bacterial, viral, and fungal infections. Upon examination, tonsils and lymph nodes are nearly undetectable and total leukocyte counts as well as serum immunoglobulin levels are abnormally low. Which of the following deficiencies is primarily responsible for the leukopenia in this patient? (A) Adenosine deaminase (B) IL-2 receptor γ -chain (CD25) (C) IL-7 receptor α-chain (CD127) (D) Janus tyrosine kinase 3 (JAK 3) (E) T box-1 (TBX1)

*The answer is A.* The patient has X-linked agammaglobulinemia, which is cause dby defective bruton tyrosine kinase (btk).

A 1-year-old male child who has experienced recurrent serious infections with staphylococcal and streptococcal bacteria is evaluated for immune deficiency during a time when he has no ongoing infection. His white blood cell count is within the normal range; however, flow cytometry analysis reveals an absence of CD19+ lymphocytes and normal numbers of CD4 and CD8+ lymphocytes. Nephelometry reveals very low levels of IgG and no measurable IgM or IgA. Regarding the above case, which molecule is defective in this disease? (A) Bruton tyrosine kinase (btk) (B) CD40 (C) CD80/86 (D) Fas ligand (E) Rag

*The answer is E.* The major clinical clues to this diagnosis include the sex of the patient and the history of serious recurrent bacterial infections. X-linked agammaglobulinemia is also known as Bruton agammaglobulinemia. The laboratory test results that support this diagnosis are the lack of CD19+ cells. The molecule CD19 is part of the B-cell coreceptor complex involved in B-cell activation. Typically, in Bruton agammaglobulinemia, T-cell numbers are normal to slightly elevated and neutrophils and monocyte numbers are normal. Immunoglobulin levels are low to absent in Bruton's as a result of the lack of B cells. Laboratory findings in acquired immunodeficiency syndrome (AIDS), adenosine deaminase (ADA) deficiency, and DiGeorge syndrome would show reduced numbers of T cells. ADA deficiency is one of the most common forms of severe combined immunodeficiency (SCID). This condition is due to a deficiency in expression of the ADA enzyme which functions in the salvage pathway of purines. Since developing, lymphocytes are particularly sensitive to ADA deficiency (inefficient in degrading dATP into 2′-deoxyadenosine), B- and T-cell numbers are impacted. DiGeorge syndrome is characterized by low to absent T-cell numbers depending on the extent of the disease. T cells can be found in patients with partial DiGeorge syndrome. AIDS is a disease resulting from an infection with human immunodeficiency virus or HIV. Generally, CD4+ T-cell numbers are significantly reduced over time and the CD4:CD8 ratio is altered. B-cell number and function are affected as these cells depend on T helper cells for responses to protein anti-gens. SLE is an autoimmune disease, not an immune deficiency disease.

A 1-year-old male child who has experienced recurrent serious infections with staphylococcal and streptococcal bacteria is evaluated for immune deficiency during a time when he has no ongoing infection. His white blood cell count is within the normal range; however, flow cytometry analysis reveals an absence of CD19+ lymphocytes and normal numbers of CD4 and CD8+ lymphocytes. Nephelometry reveals very low levels of IgG and no measurable IgM or IgA. What is the most likely diagnosis? (A) Acquired immunodeficiency syndrome (AIDS) (B) Adenosine deaminase deficiency (C) DiGeorge syndrome (D) Systemic lupus erythematosus (SLE) (E) X-linked agammaglobulinemia

*The answer is E.* This case is descriptive of hyper-IgM syndrome, a disease whose molecular defect is caused by mutations in the gene for CD40 ligand otherwise known as CD154.

A 10-month-old male with a history of recurrent, serious extracellular bacterial infections is hospitalized with pneumonia due to Pneumocystis jeroveci. An immunologic workup reveals neutropenia, normal numbers of T and B cells, normal complement levels and complement activity, elevated IgM and very low IgG and IgA. What is the molecular defect in this disease? (A) Defective expression by lymphocytes of adenosine deaminase (B) Lack of Bruton tyrosine kinase (C) Mutation of the γ-chain of the IL-2 receptor (D) Mutation of the NADPH oxidase enzyme (E) Reduced expression of CD154 on T cells

*The answer is D.* This rare syndrome is characterized by (1) recurrent infections, (2) hemorrhages secondary to thrombocytopenia, and (3) eczema. It typically manifests in boys within the first few months of life as petechiae and recurrent infections (e.g., diarrhea). It is caused by numerous distinct mutations in a gene on the X chromosome that encodes a protein called WASP (Wiskott- Aldrich syndrome protein), which is expressed at high levels in lymphocytes and megakaryocytes. WASP binds members of the Rho family of GTPases. WASP itself controls the assembly of actin filaments that are required to form microvesicles. X-linked agammaglobulinemia of Bruton (choice E) is not associated with thrombocytopenia and eczema. Choices A, B, and C are not X-linked genetic diseases. Diagnosis: Wiskott-Aldrich syndrome

A 12-month-old infant with a history of recurrent infections, eczema, generalized edema, and easy bruising is diagnosed with an X-linked, recessive, congenital immunodeficiency. The CBC shows thrombocytopenia. What is the most likely diagnosis? (A) DiGeorge syndrome (B) Isolated IgA defi ciency (C) Severe combined immunodeficiency (D) Wiskott-Aldrich syndrome (E) X-linked agammaglobulinemia of Bruton

*The answer is D.* B and T lymphocytes circulate via the vascular system to secondary lymphoid organs and tissues. Included among these tissues are lymph nodes, mucosa-associated lymphoid tissues, and spleen. In the case of lymph nodes, lymphocyte trafficking occurs through specialized postcapillary venules termed high endothelial venules (HEVs). HEVs express an array of specific cell adhesion molecules (e.g., CD31) that allow lymphocyte binding and diapedesis. The cuboidal shape of HEV cells reduces flow-mediated shear forces and specialized intercellular connections facilitate egress of lymphocytes out of the vascular space. Afferent and efferent lymphatic channels (choices A and B) do not possess HEVs. Hassall corpuscles (choice C) are found in the medulla of the thymus. Peyer patches (choice E) are organized lymphoid tissues found in the small intestine. Diagnosis: Lymphadenopathy, streptococcal pharyngitis

A 12-year-old boy presents with a 5-day history of sore throat. His temperature is 38.7°C (103°F). Physical examination reveals inflamed tonsils and swollen cervical lymph nodes. Trafficking and recirculation of blood-borne lymphocytes through the cervical lymph nodes in this patient occurs primarily at which of the following locations? (A) Afferent lymphatic vessel (B) Efferent lymphatic vessel (C) Hassall corpuscles (D) High endothelial venules (E) Peyer patches

*The answer is A.* It is likely that most or all of the other children at the day care facility have been vaccinated; thus the infant in question is less likely to be exposed to diphtheria, measles, pertussis, or polio. The remaining choices are all mechanisms by which microbes evade immune responses and would be more likely to increase the risk of infection in both the unvaccinated and vaccinated children.

A 14-month-old boy who has not received any recommended vaccines remains healthy despite his daily association with several other children for the past year at a home day care facility. Which of the following mechanisms best explains why he has not contracted diphtheria, measles, pertussis, or polio? A. Herd immunity B. Genetic drift C. Genetic shift D. Immune evasion E. Tolerance

*The answer is D.* Allergens (choice A) are agents which stimulate an IgE response and subsequent re-exposure activates mast cells to degranulate. The allergens bind to IgE molecules captured by mast cell Fc receptors and these IgE-Fc receptor complexes are long-lived providing an opportunity to become re-exposed to the antigen. Cytokine (choice C) production is a result of hapten and allergen binding but does not define the role of urushiol (poison ivy oil). Immunogens (choice E) are chemicals which elicit an immune response. This response is correct but is a broad answer to the question posed. Carriers (choice B) are usually protein in nature and associate with haptens. These carriers do not necessarily induce an immune response. Haptens are small molecules which do induce an immune response only in the presence of a carrier.

A 14-year-old girl presented with an itchy, erythematous rash following exposure to poison ivy. Which term describes the role of poison ivy oils in this response? (A) Allergen (B) Carrier (C) Cytokine (D) Hapten (E) Immunogen

*The answer is A.* Autografts are trans-plants of tissues from one part of the body to another in the same individual. These transplants are often performed on patients with burns or those with disfiguring injuries. Graft failure does occur in autologous skin grafts; however, it is not due to immunologic rejection reactions. All of the other choices are allografts and are thus prone to immunologic rejection reactions.

A 14-year-old male child with extensive third-degree burns from a house fire receives a skin graft. Skin grafted from which donor has the lowest risk of immunologic rejection? (A) The patient (B) The patient's father (C) The patient's fraternal twin brother (D) The patient's fraternal twin sister (E) The patient's mother

*The answer is A.* The AIRE (autoimmune regulator gene) codes for a transcription factor that regu-lates the expression of many genes. It is particularly active in the thymus where it is thought to allow the expression of genes encoding proteins found outside of the thymus. Thus, it enables thymic epithelial cells and dendritic cells to present extrathymic antigens in the process of central tolerance induction. A mutation in this gene would interfere with normal negative selection against self-reactive thymocytes, thereby allowing autoreactive T cells to enter the periph-eral circulation. Numerous studies have associated mutations in AIRE with the development of poly-glandular autoimmune syndrome Type-1. Genes of the complement system including C1 and C4 have been associated with systemic lupus erythematosus. A number of animal models and human genetic studies have linked polymorphisms of CTLA-4 with various autoimmune diseases including allergic encephalo-myelitis, SLE, and Type-1 diabetes. This molecule is important in the maintenance of peripheral toler-ance by down-regulating T-cell activation. FoxP3 is a transcription factor expressed by regulatory T cells. Deficiencies of FoxP3-expressing regulatory T cells have been associated with a variety of autoimmune diseases in animal models. The association of major histocompatability genes and autoimmune diseases is a strong one, and the relative risk of developing rheumatoid arthritis, Type-1 diabetes, or pamphigus vulgaris is increased in individuals expressing HLA-DR4. Surprisingly, there is no strong association of any MHC gene and the development of polyglandular autoimmune syndrome Type-1.

A 15-year-old girl is being evaluated for Addison disease. Her history is significant for hypoparathyroidism diagnosed at age 10 and recurrent bouts of mucocutaneous candidiasis since early childhood. She is diagnosed eventually with polyglandular autoimmune syndrome type-1. Which gene is associated with this condition? (A) AIRE (B) Complement C1q (C) CTLA-4 (D) FoxP3 (E) HLA-DR4

*The answer is B.* This autosomal recessive genetic disease is characterized by partial hypopigmentation of the skin, hair, and eyes, as well as immune deficiency and abnormal bleeding. This child manifested all these problems. Nonneoplastic lymphoid cell infiltration into various organs also occurs in most patients. The disease results from mutations in the gene for a protein involved in organelle protein trafficking called lysosomal-trafficking regulator protein or LYST. Any cell whose function depends on storage or secretory granules is adversely affected. Thus, melanocytes are unable to secrete melanin, leading to hypopigmentation. Neutrophils are unable to fuse lysosomes with the phagosome, resulting in defective microbicidal activity and increased infection. The function of platelets and natural killer cells, which rely on substances within granules, is impaired. Even the maturation of granule-containing cells is affected leading to the moderate decrease in neutrophils and platelets seen in this patient. Bare lymphocyte syndrome is a form of severe combined immune deficiency syndrome, mani-festing as defects in T and B cells. Hypopigmentation and defects in neutrophil and platelet functions are not seen in this disease. Thrombocytopenia purpura manifests with bleeding disorders. Wegener granulo-matosis is an immunopathologic disease involving the small- and medium-sized vessels of the respiratory tract and kidney whose manifestations include chronic sinusitis, hemoptosis, and hematuria. Wiskott-Aldrich syndrome is an X-linked recessive immune deficiency disease affecting the function of platelets, T cells, and B cells. Infants present with recurrent infections, bleed-ing disorders, and eczema.

A 2-month-old infant presents with severe pyoderma over much of her body. Her history is significant for other infections including pneumonia and for bruising very easily. Both parents are healthy with no significant medical history. Physical exam reveals an ill infant with large areas of hypopigmentation of the skin, silvery hair, and very pale eyes. Gingival inflammation and bleeding of the gums and generalized lymphadenopathy are noted. A white blood cell count reveals moderate leukopenia and thrombocytopenia. Definitive diagnosis is made by examination of a peripheral blood smear which showed unusually large granules within neutrophils and eosinophils. What is the most likely diagnosis? (A) Bare lymphocyte syndrome (B) Chediak-Higashi syndrome (C) Thrombocytopenia purpura (D) Wegener granulomatosis (E) Wiskott-Aldrich syndrome

*The answer is A.* Determination of antibody specificity occurs prior to and independent from an individual's first encounter with antigen. This process begins developmentally during prenatal and neonatal life. This process is independent of soluble factors (cytokines) produced by T cells and occurs independently of maternal immune function. By definition, memory B cells have previously encountered antigen. Somatic hypermutation occurs only after previous exposure to antigen.

A 2-year-old child exposed to an antigen for the first time already possesses a B cell with immunoglobulin specific for that antigen. This finding is best explained by A. antigen-independent immunoglobulin gene rearrangements. B. antigen stimulation of T cell cytokine production. C. maternally derived antibodies to that antigen. D. memory B cells that recognize the antigen. E. somatic hypermutation of immunoglobulins.

*The answer is B.* "Poison ivy" is a type IV hypersensitivity reaction to plants of the Rhus genus. This T-lymphocyte-mediated allergic contact dermatitis presents as urticaria and bullous eruption. Blisters rupture and heal with crusts, usually without scarring. Deposition of antigluten antibodies (choice C) occurs in patients with dermatitis herpetiformis. IgE-mediated mast cell degranulation (choice E) is part of the response to poison ivy (hypersensitivity reactions overlap), but this immediate response does not explain the pathogenesis of delayed hypersensitivity in this patient. Diagnosis: Allergic contact dermatitis

A 20-year-old gardener presents to his family physician for treatment of what he describes as "poison ivy." The patient's hands and arms appear red and are covered with oozing blisters and crusts. Which of the following best describes the pathogenesis of these skin lesions? (A) Cytotoxic antibody production (B) Delayed-type hypersensitivity (C) Deposition of antigluten antibodies (D) Deposition of circulating immune complexes (E) IgE-mediated mast cell degranulation

*The answer is E.* This type of collagen is found in the basement membranes of the lung and kidney. Type IV collagen fibers are made up of a triple helix-containing mixtures of one of the six different α-chains. The collagen epitope targeted in Goodpasture syndrome is at the carboxyl terminal of the noncollagenous domain of the α3-chain. Desmoglein-3 is a protein component of desmosomes found between keratinocytes. It is targeted by autoantibody in the disease pemphigus vulgaris. Neutrophil cytoplasmic antigens are targeted in Wegener granulomatosus. Nuclear antigens are targeted in systemic lupus erythematosus. Pathogen antigens would be found in immune complexes deposited in the kidneys in postinfectious glomerulonephritis.

A 20-year-old male was hospitalized with fever, hemoptysis, and respiratory distress. He had a 2-week history of fever, chills, cough, and shortness of breath that was unresponsive to a course of azithromycin. His cough had recently become tinged with blood. Urinalysis also showed hematuria. Blood, urine samples, and bronchoalveolar lavage samples were taken for culture and he was placed on empiric, broad-spectrum antibiotics. He quickly deteriorated, developing pulmonary hemorrhage and azotemia. Direct immunofluorescent staining of a renal biopsy is shown in the photograph. What antigen is targeted by the immune system in the above case? (A) Desmoglein-3 (B) Neutrophil cytoplasmic antigens (C) Nuclear antigens (D) Pathogen antigens (E) Type IV collagen

*The answer is A.* The patient in this case was suffering from a form of pulmonary-renal syndrome, characterized by the simultaneous occurrence of pulmonary hemorrhage and glomerulonephritis. Differential diagnosis of pulmonary-renal syndrome includes Goodpasture syndrome, systemic lupus erythematosus (SLE), and Wegener granulomatosis. The smooth, uniform pattern of fluorescence seen on the biopsy is typical for Goodpasture syndrome. A more heterogeneous fluorescence, often called "lumpy bumpy," is typical for immune complex-mediated glomerulonephritis as occurs in SLE. Postinfectious glomerulonephritis is also due to immune complex deposition and would appear similarly; this disease is not part of the pulmonary-renal syndrome. Kidney biopsies from patients with IgA nephropathy show IgA deposition in the mesangium, while kidney biopsies from Wegener granulomatosus show little or no deposition of IgG or complement, and thus weak or no fluorescence.

A 20-year-old male was hospitalized with fever, hemoptysis, and respiratory distress. He had a 2-week history of fever, chills, cough, and shortness of breath that was unresponsive to a course of azithromycin. His cough had recently become tinged with blood. Urinalysis also showed hematuria. Blood, urine samples, and bronchoalveolar lavage samples were taken for culture and he was placed on empiric, broad-spectrum antibiotics. He quickly deteriorated, developing pulmonary hemorrhage and azotemia. Direct immunofluorescent staining of a renal biopsy is shown in the photograph. What is the diagnosis? (A) Goodpasture syndrome (B) IgA nephropathy (C) Postinfectious glomerulonephritis (D) Systemic lupus erythematosus (E) Wegener granulomatosis

*The answer is B.* This patient is having a hypersensitivity type I reaction, which is caused by the binding of antigen to preexisting cell-fixed IgE antibodies.

A 20-year-old woman is brought to the emergency department 20 minutes after being stung by a wasp. She says that she feels a lump in her throat and chest tightness. She has a history of allergy to wasp venom. Her pulse is 120/min, and blood pressure is 80/40 mm Hg. Physical examination shows eruptions that coalesce into giant urticaria. There is audible wheezing. Which of the following best describes the cause of this patient's reaction? (A) Activation of macrophages by soluble immune complexes (B) Binding of antigen to preexisting cell-fixed IgE antibodies (C) Formation of IgG antibodies against extracellular matrix antigen (D) Formation of IgM antibodies against cell surface receptor antigens (E) Induction of a cytotoxic reaction by CD8+ T lymphocytes

*The answer is C.* Immediate-type hypersensitivity is manifested by a localized or generalized reaction that occurs within minutes after exposure to an antigen or "allergen" to which the person has previously been sensitized. In its generalized and most severe form, immediate hypersensitivity reactions are associated with bronchoconstriction, airway obstruction, and circulatory collapse, as seen in anaphylactic shock. Type I hypersensitivity reactions feature the formation of IgE antibodies that bind avidly to Fc-epsilon (Fc-ε) receptors on mast cells and basophils. The high-avidity binding of IgE accounts for the term cytophilic antibody. Once exposed to a specific allergen that has resulted in the formation of IgE, a person is sensitized. Subsequent responses to the allergen induce an immediate release of a cascade of proinflammatory mediators. These mediators are responsible for smooth muscle contraction, edema formation, and the recruitment of eosinophils. None of the other immunoglobulin classes mediates immediate hypersensitivity. Diagnosis: Asthma

A 20-year-old woman with a history of asthma and allergies undergoes skin testing to identify potential allergens in her environment. A positive skin reaction to ragweed in this patient would be mediated by which of the following classes of immunoglobulin? (A) IgA (B) IgD (C) IgE (D) IgG (E) IgM

*The answer is E.* This case is descriptive of Wiscott-Aldrich syndrome (WAS). This disease classically presents with increased susceptibility to infection, eczema, and thrombocytopenia. The finding of unusually small platelets, as indicated by the decreased mean platelet volume, is particularly helpful in making a diagnosis of this condition. In addition, low IgM and IgG are characteristic of this disease, as is the lack of isohemagglutinins. Elevations in IgA and IgE are often found. WAS is an X-linked recessive disease due to a mutation in the WAS protein or WASP. This pro-tein, found in leukocytes and megakaryocytes, binds to actin and is involved in its reorganization. Actin polymerization and depolymerization are essential for the normal functioning of cells. In T lymphocytes for instance, actin reorganization follows engagement of the T-cell receptor to antigen presented on MHC molecules and results in the clustering of relevant receptors and their ligands to one pole of the cell. This clustering of receptors and ligands holds the two cells together to form the immunologic synapse across which cytokines can be exchanged between the T cell and the antigen-presenting cell. Actin reorganization is also essential for the movement of thymocytes as they mature within the thymus, the migration of mature lymphocytes into lymph nodes and sites of infection, as well as in cell division. Mitogen stimulation of lymphocytes often reveals decreased responsiveness. Although lymphocyte numbers are relatively normal early in life, patients with WAS develop lymphopenia with time. Bone marrow stem cell transplantation is the treatment of choice for this condition.

A 21-month-old male child is being evaluated for recurrent otitis media that returns rapidly follow-ing completion of an antibiotic regimen. His history is significant for pneumonia at 14 months of age. Physical exam reveals eczema and a petechial rash. His mother reports he is prone to nose bleeds. A complete blood count reveals normal white blood cell number and distribution, thrombocytopenia, and decreased mean platelet volume. Immunoglobulin levels show decreased IgM and IgG, but elevated IgA and IgE. Isohemagglutinins were not detected even though the child's blood type was O. What is the cellular defect characteristic of this condition? (A) Defective expression of chemokine receptors (B) Defective expression of costimulatory molecules on T cells (C) Defective expression of signal transduction molecules (D) Defective maturation of stem cells into white blood cells (E) Defective polymerization and depolymerization of actin

*The answer is B.* Systemic lupus erythematosus (SLE) is an autoimmune, inflammatory disease that may involve almost any organ but characteristically affects the kidneys, joints, serous membranes, and skin. Autoantibodies are formed against a variety of self-antigens. The most important diagnostic autoantibodies are those against nuclear antigens—in particular, antibody to double-stranded DNA and to a soluble nuclear antigen complex that is part of the spliceosome and is termed Sm (Smith) antigen. High titers of these two autoantibodies (termed antinuclear antibodies) are nearly pathognomonic for SLE. Antibodies to rheumatoid factor (choice D) are seen in patients with rheumatoid arthritis. Antineutrophil cytoplasmic antibodies (choices A and C) are seen in patients with small vessel vasculitis (e.g., Wegener granulomatosis). Diagnosis: Systemic lupus erythematosus

A 21-year-old woman presents with a 3-month history of malaise, joint pain, weight loss, and sporadic fever. The patient appears agitated. Her temperature is 38°C (101°F). Other physical findings include malar rash, erythematous-pink plaques with telangiectatic vessels, oral ulcers, and nonblanching purpuric papules on her legs. Laboratory studies show elevated levels of blood urea nitrogen and creatinine. Antibodies directed to which of the following antigens would be expected in the serum of this patient? (A) C-ANCA (anti-proteinase-3) (B) Double-stranded DNA (C) P-ANCA (anti-myeloperoxidase) (D) Rheumatoid factor (E) Scl-70 (anti-topoisomerase I)

*The answer is A.* Acquired deficiencies of early complement components occur in patients with autoimmune diseases, especially those associated with circulating immune complexes (e.g., systemic lupus erythematosus [SLE]). Antigen-antibody complexes formed in the circulation during the active stage of these diseases lead to a marked reduction in circulating levels of complement proteins (hypocomplementemia). None of the other choices mediates hypocomplementemia in patients with SLE. Diagnosis: Systemic lupus erythematosus

A 21-year-old woman presents with a 3-month history of malaise, joint pain, weight loss, and sporadic fever. The patient appears agitated. Her temperature is 38°C (101°F). Other physical findings include malar rash, erythematous-pink plaques with telangiectatic vessels, oral ulcers, and nonblanching purpuric papules on her legs. Laboratory studies show elevated levels of blood urea nitrogen and creatinine. Serum levels of complement proteins may be reduced during the active phase of the disease in the patient described due to which of the following mechanisms of disease? (A) Binding of complement to immune complexes (B) Decreased complement protein biosynthesis (C) Defective activation of the complement cascade (D) Increased urinary excretion of immunoglobulins (E) Stimulation of the acute phase response

*The answer is A.* This case is descriptive of a new onset of systemic lupus erythematosus (SLE). Patients with SLE make a variety of autoantibodies, particularly antinuclear antibodies. These autoantibodies are also found in other immune disorders of connective tissue, thus, the finding of antinuclear antibodies is suggestive of, but not diagnostic of SLE. It is considered a screening test for SLE. Antibodies to double-stranded DNA are unique to SLE, and are found in about 60% of patients with this disease. Anti-Sm antibodies are even more specific for SLE. All the other test results listed could be positive in a patient with SLE, but do not confirm the diagnosis. Decreased serum complement levels are seen in diseases marked by complement activa-tion, such as occurs in the Type III hypersensitivity reactions that are at the heart of the pathology of SLE. Elevated C-reactive protein is a nonspecific marker of systemic inflammation, similar to elevated erythro-cyte sedimentation rates. Hemolytic anemia, indicated by a positive Coombs test, and other immune-mediated cytopenias can be seen in patients with SLE who develop antibodies to blood cells.

A 22-year-old female presents with an erythematous malar rash as seen in the accompanying photograph after spending spring break on the beach. Prior to this incident, she had been feeling increasingly fatigued and noticed painful swelling in her hands and wrists. Which of the following test results would yield a definitive diagnosis of her condition? (A) Antibodies to double-stranded DNA (B) Decreased serum complement levels (C) Elevated antinuclear antibody levels (D) Elevated C-reactive protein (E) Positive Coombs test

*The answer is B.* This person is experiencing a type I hypersensitivity reaction. In type 1 hypersensitivity, B-cells are stimulated (by CD4+TH2 cells) to produce IgE antibodies specific to an antigen, which activiates mast cells.

A 22-year-old man is brought to the emergency department in respiratory distress 15 minutes after he was stung on the arm by a wasp. His pulse is 100/min, respirations are 30/min, and blood pressure is 100/60 mm Hg. Physical examination shows grunting respirations and subcostal retractions. Expiratory wheezes are heard over both lung fields. There is generalized urticaria. Secretion of the molecule causing this patient's symptoms is most likely mediated by which of the following? (A) Activation of complement (B) Activation of mast cell (C) Activation of T lymphocytes (D) Production of IgA (E) Production of IgG (F) Production of IgM

*The answer is A.* This case is descriptive of hereditary angioedema, a condition attributed to a decrease in or dysfunction of C1 inhibitor (C1-INH). This protein is a multifunction serine protease normally found in large amounts in serum. It binds to and inactivates C1r and C1s. It also inactivates proteins of the clotting and kinin systems. Decreased amounts of this protein result in subcutaneous and/ or submucosal edema through compromised inactivation of the complement and/or the kinin system. Hereditary angioedema is an autosomal dominant trait. Deficiencies in the other complement components listed do not lead to hereditary angioedema

A 24-year-old woman presents with airway obstruction secondary to laryngeal swelling. Her history is significant for episodes of recurrent facial swelling that began sometime in childhood. The swelling, which often involves the lips, begins suddenly without recognizable precipitating events and lasts for 2 to 3 days. Laboratory tests reveal a deficiency involving the complement system. Which component of the complement system is deficient in this woman? (A) C1 inhibitor (B) C3 convertase (C) C5 convertase (D) Mannose-binding lectin (E) Membrane attack complex

*The answer is A.* The advent of transplantation of bone marrow into patients whose immune system has been ablated or into otherwise immunodeficient patients has resulted in the complication of graft-versus-host disease (GVHD). GVHD occurs when lymphocytes in the grafted tissue recognize and react to the recipient. GVHD can also occur when an immunodeficient patient is transfused with blood containing HLA-incompatible lymphocytes. The major organs affected in GVHD include the skin, gastrointestinal tract, and liver. Clinically, GVHD manifests as rash, diarrhea, abdominal cramps, anemia, and liver dysfunction. None of the other cells mediates GVHD. Diagnosis: Graft-versus-host disease

A 24-year-old woman with leukemia receives an allogeneic bone marrow transplant. Three weeks later, she develops a skin rash and diarrhea. Liver function tests show elevated serum levels of AST and ALT. A skin biopsy discloses a sparse lymphocytic infiltrate in the dermis and epidermis, as well as apoptotic cells in the epidermal basal cell layer. Skin rash and diarrhea in this patient are caused primarily by which of the following cells? (A) Donor lymphocytes (B) Donor plasma cells (C) Fixed tissue macrophages (D) Recipient lymphocytes (E) Recipient plasma cells

*The answer is E.* The patient in this case is experiencing shock. A number of clinical syndromes can be responsible for these symptoms including trauma, septic shock, dehydration, and heart damage. This patient, however, experienced these conditions suddenly upon consumption of food. Thus, this is a case of systemic anaphylaxis, possible due to exposure to peanut antigens in the food he ingested. Clues to this diagnosis include the extremely low blood pressure and other physical signs. Cold and clammy skin is the classic sign of impending shock. It suggests that the patient is not delivering suffucient blood to the peripheral tissues. The low blood pressure is a result of increased vascular permeability and vasodilation secondary to wide-spread mast cell and basophil degranulation. Mediators from these cells are responsible for the development of urticaria and bronchoconstriction leading to dyspnea and wheezing. Since total respiratory collapse could occur in minutes, urgent therapeutic intervention is required. Epinephrine, given either subcutaneously or intramuscularly, is the drug of choice for rapid reversal of hypotension, wheezing, and dyspnea in anaphylaxis. An inhaled β-agonist is also effective for bronchospasm, but would not have an effect on hypotension. β-blockers are used to treat hypertension, and may interfere with the actions of epinephrine in patients taking these drugs. Corticosteroids are used to treat anaphylaxis; however, their protective effects are slow to develop, and they are not used to reverse hypo-tension. Likewise, antihistamines such as diphenhydramine are used in anaphylaxis, primarily to reverse urticaria and other cutaneous manifestations.

A 25-year-old male is brought by ambulance to the emergency department after collapsing at a restaurant. His history is significant for peanut allergy. Physical exam reveals a blood pressure of 50/30 mm Hg, cold, clammy skin, diffuse urticaria, dyspnea, and wheezing. What drug is best to rapidly reverse the man's symptoms? (A) Inhaled β-agonist (B) Intravenous β-blocker (C) Intravenous methylprednisolone (D) Oral diphenhydramine (E) Parenteral epinephrine

*The answer is E.* One third of patients with systemic lupus erythematosus (SLE) possess elevated concentrations of antiphospholipid antibodies. This phenomenon predisposes these patients to thromboembolic complications, including stroke, pulmonary embolism, deep venous thrombosis, and portal vein thrombosis. The clinical course of SLE is highly variable and typically exhibits exacerbations and remissions. With the recognition of mild forms of the disease, improved antihypertensive medications, and the use of immunosuppressive agents, the overall 10-year survival rate approaches 90. Antibodies against clotting factors (choice C) or fibrinolytic enzymes (choice D) are not involved in the clotting tendency associated with SLE. Diagnosis: Systemic lupus erythematosus%

A 25-year-old woman complains of low-grade fever, fatigue, and persistent rash over her nose and upper chest. She also notes pain in her knees and elbows. A skin biopsy shows dermal inflammation and granular deposits of IgG and C3 complement along the basement membrane at the epidermal/dermal junction. Urinalysis reveals microscopic hematuria and proteinuria. The antinuclear antibody test is positive. The development of thromboembolic complications (e.g., deep venous thrombosis) in this patient is commonly associated with elevated serum levels of antibodies to which of the following antigens? (A) ABO blood group antigens (B) Class II HLA molecules (C) Clotting factors (D) Fibrinolytic enzymes (E) Phospholipids

*The answer is D.* IL-2 causes an increase in CD4+ T lymphocytes.

A 28-year-old man with a history of intravenous drug use comes to the physician because of a 6-week history of fever, nonproductive cough, chills, and progressive shortness of breath. His temperature is 39°C (102.2°F), pulse is 110/min, respirations are 32/min and regular, and blood pressure is 120/80 mm Hg. Physical examination shows a white, patchy, loosely adherent exudate on the buccal mucosa bilaterally. A chest x-ray shows bilateral interstitial infiltrates. After receiving treatment for pneumonia, he agrees to participate in a clinical study of the effects of interleukin-2 (IL-2). After administration of IL-2, which of the following hematologic changes is most likely in this patient? (A) Decreased CD4+ T lymphocytes (B) Decreased erythrocytes (C) Decreased platelet count (D) Increased CD4+ T lymphocytes (E) Increased erythrocytes (F) Increased platelet count

*The answer is A.* Myasthenia gravis is a type II hypersensitivity disorder caused by antibodies that bind to the acetylcholine receptor. These antibodies interfere with the transmission of neural impulses at the neuromuscular junction, causing muscle weakness and easy fatigability. External ocular and eyelid muscles are most often affected, but the disease is often progressive and may cause death by respiratory muscle paralysis. Autoantibodies to desmoglein-3 (choice C) are found in patients with pemphigus vulgaris, an autoimmune blistering skin disorder. Antibodies to the TSH receptor (choice E) are seen in patients with Graves hyperthyroidism. Antibodies to calcium channels (choice B) are found in patients with Eaton-Lambert syndrome. This paraneoplastic syndrome also manifests as muscle weakness but is usually associated with small cell carcinoma of the lung. Rheumatoid factor (choice D) represents multiple antibodies directed against the Fc portion of IgG and is seen in patients with rheumatoid arthritis and many other collagen vascular diseases. Diagnosis: Myasthenia gravis, thymoma

A 30-year-old woman complains of impaired speech and frequent aspiration of food. Physical examination reveals diplopia and drooping eyelids. A mediastinal mass is removed and diagnosed as thymoma. The symptoms of muscle weakness in this patient are caused by antibodies directed against which of the following cellular components? (A) Acetylcholine receptor (B) Calcium channel (C) Desmoglein-3 (D) Rheumatoid factor (E) Thyroid-stimulating hormone (TSH) receptor

*The answer is D.* Selective IgA deficiency is the most common primary immunodeficiency syndrome, with an incidence of 1:700 among Europeans. Although patients are often asymptomatic, they occasionally present with respiratory or gastrointestinal infections of varying severity. They also display a strong predilection for allergies and collagen vascular diseases. Patients with IgA deficiency have normal numbers of IgA-bearing B cells, and their varied defects result in an inability to synthesize and secrete IgA subclasses. Patients with chronic mucocutaneous candidiasis (choice B) show an increased susceptibility to Candida infections and also may exhibit various endocrine disorders (e.g., hypoparathyroidism and Addison disease). The other choices are associated with severe immunodeficiency. Diagnosis: Selective IgA deficiency

A 30-year-old woman is found to have a congenital immunodeficiency that has remained largely asymptomatic throughout her life. Which of the following is the most likely diagnosis? (A) Adenosine deaminase defi ciency (B) Chronic mucocutaneous candidiasis (C) Purine nucleoside phosphorylase defi ciency (D) Selective IgA deficiency (E) Wiskott-Aldrich syndrome

*The answer is B.* The relentless progression of HIV infection is now recognized as a continuum that extends from an initial asymptomatic state to the immune depletion that characterizes patients with overt AIDS. The fundamental lesion is infection of CD4+ (helper) T lymphocytes, which leads to the depletion of this cell population and impaired immune function. As a result, patients with AIDS usually die of opportunistic infections. HIV does infect the monocyte/macrophage lineage (choice D), but infected cells exhibit little if any cytotoxicity. NK cell activity (choice E) is also decreased in AIDS. This defect may contribute to the appearance of malignant tumors and the viral infections that plague these patients. The suppression of NK cell activity has been related to a decrease in the number of NK cells and to a reduction in IL-2 levels due to the loss of CD4+ cells.

A 31-year-old man with AIDS complains of difficulty swallowing. Examination of his oral cavity demonstrates whitish membranes covering much of his tongue and palate. Endoscopy also reveals several whitish, ulcerated lesions in the esophagus. These pathologic findings are fundamentally caused by loss of which of the following immune cells in this patient? (A) B lymphocytes (B) Helper T lymphocytes (C) Killer T lymphocytes (D) Monocytes/macrophages (E) Natural killer (NK) cells

*The answer is A.* Rhogam is a preparation of anti-Rh antibodies used to prevent sensitization of an Rh-negative mother by Rh-positive fetal red blood cells. These antibodies cause hemolysis of fetal cells that enter the maternal circulation, destroying the cells before the Rh antigens have a chance to be presented to the mother's immune system and stimulate antibody production. An Rh-negative mother should receive this preparation after the birth of each Rh-positive child. Intravenous immune globulin can be given to affected infants after birth to help prevent destruction of red cells by maternal anti Rh antibodies that crossed the placenta during gestation. Plasmaphoresis is not used in the prevention or management of this disease. Transfusion of the mother with Rh-positive blood before pregnancy or after delivery will stimulate her immune system to make anti-Rh antibodies, thereby promoting, not preventing, hemolytic disease in any Rh-positive fetuses that she carries.

A 33-year-old woman is pregnant with her second child. Her blood type is B, Rh-negative, and her husband's is AB, Rh-positive. Her first child has type B blood and expresses the D antigen in his red blood cells. How is hemolytic disease of the newborn prevented? (A) Administration of anti-Rh antibodies to Rh-negative women after the birth of each Rh-positive child (B) Infusion of intravenous immune globulin to Rh-negative women during each pregnancy (C) Periodic plasmaphoresis of Rh-negative women during pregnancy (D) Transfusion of Rh-negative women with Rh-positive blood after delivery of each child (E) Transfusion of Rh-negative women with Rh-positive blood prior to each pregnancy

*The answer is D.* Hemolytic disease of the newborn results from Rh incompatibility between the mother and the fetus. It can occur when an Rh-negative mother is carrying an Rh-positve fetus. Fetal blood mixes with maternal blood during gestation or during the birth process. The mother becomes sensitized to the Rh antigens and makes anti-Rh antibodies. In a subse-quent pregnancy with an Rh-positive fetus, these IgG anti-Rh antibodies can cross the placenta and cause hemolysis of the fetal red blood cells. Sensitization of the mother to fetal Rh antigens can be detected by the indirect antiglobulin test, also called the indirect Coombs test. In this test, the mother's serum is mixed with Rh-positive red blood cells. If anti-Rh antibodies are present in maternal serum, they will bind to the Rh-positive red blood cells. Rabbit antihuman globulin, made by injecting rabbits with the γ -globulin fraction of human serum, is added to the mixture, and the anti-Rh antibody-coated red cells agglutinate, giving a positive reaction. This test should be done at the first prenatal visit. A direct Coombs test done on the mother's red cells would indicate, if positive, that her cells were attacked by antired cell antibodies. ABO incompatibility between the mother and fetus does not lead to hydrops fetalis, or hemolytic disease of the newborn. Testing for fetal bilirubin levels and ultra-sound studies are methods of assessing the severity of hemolysis in an affected fetus.

A 33-year-old woman is pregnant with her second child. Her blood type is B, Rh-negative, and her husband's is AB, Rh-positive. Her first child has type B blood and expresses the D antigen in his red blood cells. What test can be used to determine if her fetus is at risk for hemolytic disease of the newborn? (A) ABO blood typing of the fetus prior to birth (B) Bilirubin level in umbilical cord blood (C) Direct Coombs test on the mother's red cells (D) Indirect antiglobulin test on the mother's serum (E) Ultrasound

*The answer is E.* The reaction to PPD in a tuberculosis patient is the result of Mycobacterium tuberculosis-specific memory T helper 1 (Th1 cells that encounter the antigen presented by skin dendritic cells (Langerhans cells). Following antigen interaction, the Th1 cell releases cytokines and chemokines, which recruit and activate mononuclear cells from the blood. The inflammatory response that develops, with its vasodilation and cellular infiltrate, accounts for the erythema and induration seen at the site of antigen injection. If any of the other cells were primarily involved in this reaction, even uninfected individuals would respond by mounting an inflammatory response following antigen injection. The skin test is used to detect memory cells elicited following primary exposure to M. tuberculosis.

A 34-year-old inmate presented to the prison clinic with fever, cough, and night sweats. Tuberculosis was suspected and PPD (purified protein derivative) was injected intradermally. A skin reaction characterized by erythema and induration developed at the injection site within 2 days. Which cell type is primarily responsible for reacting to the PPD and releasing mediators that resulted in the skin manifestation? (A) Endothelial cells (B) Keratinocytes (C) Langerhans cells (D) Mast cells (E) Th1 cells

*The answer is D.* Although the incorrect choices may cause eye irritation, seasonal conjunctivitis is typically caused by allergies to pollens that are released during a particular time of the year. Allergic rhinitis (hay fever) is the most common type I hypersensitivity disease in adults. It may be caused by pollen, house dust, animal dandruff, and many other allergens. Antigens inhaled react with the IgE attached to basophils in the nasal mucosa, thereby triggering the release of vasoactive substances stored in cytoplasmic granules. Histamine, the main mediator released from mast cells, increases the permeability of mucosal vessels, causing edema and sneezing. Diagnosis: Conjunctivitis, hypersensitivity reaction

A 35-year-old man asks for advice regarding seasonal eye itching and runny nose. Recurrent conjunctivitis in this patient is most likely caused by which of the following mechanisms of disease? (A) Autoimmunity (B) Bacterial infection (C) Chemical toxicity (D) Hypersensitivity (E) Viral infection

*The answer is C.* Myasthenia gravis is characterized by drooping of her eyelids and muscle weakness that improves with rest.

A 36-year-old female is diagnosed with myasthenia gravis. What were her most likely clinical manifestations? (A) Abdominal pain, weight loss, and bloody diarrhea (B) Ascending weakness and paresthesia (C) Drooping of her eyelids and muscle weakness that improves with rest (D) Fatigue, pallor, and mild jaundice (E) Goiter, palpitations, and proptosis of the eyes

*The answer is E.* This case is descriptive of psoriasis, an inflammatory skin disease characterized by excessive proliferation of keratinocytes. Several dif-ferent forms of the disease exist. This individual has plaque psoriasis. In addition to skin lesions, individuals frequently have arthritis and nail problems. The pathogenesis of psoriasis is uncertain; however, recent evidence supports a role for T cells. It is believed that dendritic cells present a skin antigen to T cells which than migrate as effector cells into the skin where they secrete cytokines which stimulate keratinocyte pro-liferation. One key cytokine involved in psoriasis is tumor necrosis factor. In fact, inhibitors of tumor necrosis factor have been used with success in this disease. Cyclosporine is a drug used to prevent trans-plant rejection. It works by blocking the calcineurin-mediated activation of a transcription factor necessary for the expression of IL-2 and other cytokines by T cells. Thus, T-cell activation is inhibited. The efficacy of cyclosporine in reducing psoriasis lesions was one of the first pieces of evidence suggesting that T cells play a role in the pathogenesis of this disease. There are many different pharmacologic approaches to the treatment of this disease, but none of them lead to complete resolu-tion of symptoms.

A 36-year-old male has been treated for 10 years for the scaly, plaquelike lesions shown in the accompanying photograph. The plaques are widespread on his body and have been resistant to several different therapies. His physician prescribes cyclosporine that leads to a rapid reduction in lesions. Which cell is implicated in the pathogenesis of this disease as demonstrated by the response to this drug? (A) B cells (B) Dendritic cells (C) Eosinophils (D) Neutrophils (E) T cells

*The answer is C.* This is an allergic response to the yellow jacket venom involving mast cell degranulation. As a consequence of degranulation, histamine is released. Complement proteins (choice A) are a set of serum proteins which bind to microbial surfaces in order to promote opsonization and cell killing through formation of the membrane attack complex. IgG is an antibody isotype which can initiate complement activation and serve to promote opsonization of pathogens. TNF or tumor necrosis factor is a proinflammatory cytokine released by phagocytes and activated T cells, which elicits inflammatory activity. Norepinephrine is a hormone growth factor that plays a role during stress responses but is not responsible for allergic reaction.

A 36-year-old woman with severe allergy to yellow jackets was stung multiple times at a soccer game. Within minutes she developed respiratory distress and became unconscious. Which mediator is primarily responsible for this reaction? (A) Complement (B) IgG (C) Histamine (D) TNF (E) Norepinephrine

*The answer is C.* This case is descriptive of leukocyte adhesion deficiency, a diagnosis strongly suggested by the lack of pus formation in the current infection and confirmed by the lack of β2-integrins on leukocytes. β2-integrins are membrane-associated protein heterodimers consisting of CD18 and one of the families of CD11 glycoproteins. Leukocyte function antigen-1 (LFA-1) is comprised of CD18 and CD11a, and is found on leukocytes; whereas the expression of CD11b/CD18 and CD11c/CD18 is restricted to myeloid cells. These molecules interact with ligands on endothelial cells during inflammation, and a lack of expression prohibits the migration of leukocytes into areas of infection. Pus formation is evidence of an acute inflammatory response. The lack of pus formation in this child indicates an inability of the neutrophils to bind to endothelium and migrate into tissues in response to infection. β2 -integrins are also important in other cell-to-cell interactions including the interaction of T cells with antigen-presenting cells. CD18 forms a part of the complement receptor on phagocytes, thus its deficiency impairs phagocytosis. The association of deficient β2-integrin expression delayed umbilical cord separation, and poor wound healing is poorly understood. The other immunologic processes listed are not affected in this disease.

A 4-month-old girl child is hospitalized with a serious staphylococcal skin infection without pus formation. Her history is significant for delayed separation of the umbilical cord, recurrent infections, gingivitis, and poor wound healing. Her CBC and differential shows marked leukocytosis; however, flow cytometry studies showed a lack of expression of β2-integrins on leukocytes. What immunologic problem would this lead to? (A) Failure of lymphopoiesis (B) Reduced antigen processing by dendritic cells (C) Reduced diapedesis of neutrophils (D) Reduced expression of tolllike receptors (E) Reduced angiogenesis

*The answer is C.* Allergens such as dust, animal dander, and pollen bind to IgE expressed on the surface of mast cells. Such IgE is produced during the first exposure to the allergen and this IgE binds to high-affinity Fc receptors on mast cells. This is known as sensitization. Secondary encounter with the allergen results in binding of the antigen to the IgE and activation of the mast cells (degranulation). Macrophages (choice B) play a role in type IV delayed-type hypersensitivity. B cells (choice A) produce the initial antibodies (IgE) to the allergen but do not release vasoactive amines as does mast cells. Th1 cells (choice D) are the mediators of type IV hyper-sensitivity and Th2 cells (choice E) do promote the IgE production but again are not responsible for the mediators.

A 4-year-old child has atopic dermatitis due to severe allergies to dust, animal dander, and many kinds of pollens. Mediators released from which cell type are responsible for the clinical manifestations immediately following exposure to these substances? (A) B cells (B) Macrophages (C) Mast cells (D) Th1 cells (E) Th2 cells

*The answer is A.* This case describes a child with chronic granulomatous disease. The key clinical clue to this diagnosis is the recurrent serious bacterial infections. The key lab result is the NBT test which assesses the capacity of phagocytes to produce superoxides by NADPH oxidase. These superoxides can then be converted to hydrogen peroxide and hydroxyl radicals. A lack of NBT reduction (i.e., a negative test result) indicates a deficiency in the enzyme. While endotoxin (choice B) can increase WBC counts and IgG levels, the NBT test will be positive in this case. Accordingly, HIV reactivation (choice C), CD132 (choice D), or ADA (choice E) deficiencies will result in lowered WBC counts and serum IgG levels (CD132 and ADA). Consequently, CD132 or ADA deficiencies are indicative of severe combined immunodeficiency.

A 4-year-old male presents with a history of serious, recurrent bacterial infections including pneumonia, sepsis, and perianal abscesses. Physical exam reveals hepatosplenomegaly. An immune deficiency disease is suspected and his workup reveals WBC counts and serum IgG levels in the high normal range. The nitroblue tetrazolium (NBT) test indicated that the patient's neutrophils did not reduce NBT dye. Which of the following best explains these results? (A) Deficiency in NADH or NADPH oxidase (B) Presence of endotoxin in the blood (C) Human immunodeficiency virus reactivation (D) Deficiency in the common γ -chain (CD132) (E) Deficiency in adenosine deaminase

*The answer is C.* Class I molecules of the major histocompatibility complex present foreign peptides and are recognized by cytotoxic T lymphocytes during graft rejection or during cell-mediated killing of virus-infected cells. All tissues express class I molecules, whereas class II molecules (choice D) are displayed primarily on macrophages and B lymphocytes. CD4 and CD8 (choices A and B) are cell surface markers of helper and killer T lymphocytes, respectively. GlyCAM-1 (choice E) facilitates lymphocyte recirculation by providing a receptor for leukocyte attachment to high endothelial venules. Diagnosis: Chronic hepatitis B

A 40-year-old man complains of having yellow skin and sclerae, abdominal tenderness, and dark urine. Physical examination reveals jaundice and mild hepatomegaly. Laboratory studies demonstrate elevated serum bilirubin (3.1 mg/dL), decreased serum albumin (2.5 g/dL), and prolonged prothrombin time (17 seconds). Serologic tests reveal antibodies to hepatitis B core antigen (IgG anti-HBcAg). The serum is also positive for HBsAg and HBeAg. What glycoprotein on virally infected hepatocytes provides a target for cell-mediated cytotoxicity in the patient described? (A) CD4 (B) CD8 (C) Class I HLA molecules (D) Class II HLA molecules (E) GlyCAM-1

*The answer is E.* The clinicopathologic findings presented here indicate that this patient is a chronic HBV carrier with active hepatitis. Humoral immune responses to specific viral antigens in this patient involve the activation and differentiation of B lymphocytes into antibody-secreting plasma cells. Analogous to T cells, B cells express an antigen-binding receptor, namely mIg. This immunoglobulin bears the same antigen specificity as the soluble immunoglobulin that is ultimately secreted. Class I HLA molecules (choice C) provide targets for CD8+ T cells in cell-mediated cytotoxicity. Class II HLA molecules (choice D) are recognized by CD4+ T cells, which become activated to synthesize an array of cytokines. Diagnosis: Humoral immunity, chronic hepatitis

A 40-year-old man complains of having yellow skin and sclerae, abdominal tenderness, and dark urine. Physical examination reveals jaundice and mild hepatomegaly. Laboratory studies demonstrate elevated serum bilirubin (3.1 mg/dL), decreased serum albumin (2.5 g/dL), and prolonged prothrombin time (17 seconds). Serologic tests reveal antibodies to hepatitis B core antigen (IgG anti-HBcAg). The serum is also positive for HBsAg and HBeAg. Which of the following glycoproteins serves as the principal cell surface receptor for viral antigens on B lymphocytes in this patient? (A) CD4 (B) CD8 (C) HLA class I molecules (D) HLA class II molecules (E) Membrane immunoglobulin

*The answer is D.* Human and animal studies suggest that these cytokines are present in the brain during times of dis-ease remission and act to regulate Th1 cells. The other cytokines listed are not associated with MS remission, and many (CC-chemokines, γ-interferon, TNF-α, IL-12, and IL-17) are associated with disease progression. The recruitment of Th1 cells into the CNS would accelerate the disease. Proliferation of oligodendrocytes is not promoted by γ -IFN or TNF-α. T cells secrete matrix metalloproteases in patients with MS. Activation of endothelial cells and memory T cells is not consistent with disease remission.

A 43-year-old woman has a diagnosis of relapsing and remitting multiple sclerosis. Which cytokines pre-dominate in the CNS during periods of remission and what is their effect? (A) CC-chemokines/recruit Th1 cells into CNS (B) γ -Interferon and tumor necrosis factor-α (TNF-α)/support proliferation of oligodendrocytes (C) IL-2, IL-5, TNF-α/activate microglia and astrocytes to secrete matrix metalloproteases (D) IL-4, IL-10, TGF-β/downregulate Th1 cells (E) IL-17 and TNF-α/activate endothelial cells and memory T cells

*The answer is E.* Intradermal inoculation of candidal or mumps antigens in immune competent individuals results in an infiltration of mononuclear cells at the inoculation after 48 hours. This reaction is due to the activation of memory Th1 cells by antigen presenting macrophages and dendritic cells in the skin. Activated memory Th1 cells release cytokines that recruit mononuclear cells to the area. This reaction requires about 48 hours to develop and results in the induration and erythema characteristic of a positive delayed type hypersensitivity (DTH) reaction. The inability to carry out a DTH reaction to commonly encountered antigens such as Candida and mumps virus indicates a dysfunction of Th1 cells. Antiretroviral drugs do not interfere with this type of test. Neutropenia can lead to uncontrolled candidal infections but it does not result in skin test anergy. Antibodies and CD8 cells do not play a role in the DTH reaction.

A 45-year-old AIDS patient was enrolled in a drug development study. As part of the study, he underwent anergy skin testing. He was inoculated intradermally with candidal and mumps virus antigens. No reaction was demonstrable after 48 h. What is the most likely explanation for this? (A) Antiretroviral drugs are known to interfere with this type of test (B) He has neutropenia and cannot control the growth of Candida (C) He lacks antibodies to both Candida and mumps virus (D) His CD8 cells are not being activated due to decreased CD4 cell numbers (E) His Th1 cells are unable to carry out a delayed type hypersensitivity reaction

*The answer is D.* Platelets and erythrocytes serve as blood components and do not generate immune responses against parasites (choices A and B). Neutrophils (choice C) are surveillance cells which may engage the parasite but the most effective response is generated by eosinophils which express Fc epsilon receptors. These receptors allow the eosinophils to bind to IgE directed against parasitic microbes. Such binding will result in antibody-dependent cell cytoxicity or ADCC. Monocytes (choice E) are bone marrow and circulating blood cells which can differentiate into macrophages and dendritic cells. However, these cells would not be as effective as the eosinophils in killing the parasite.

A 45-year-old female presents with anorexia and some abdominal pain. Fecal smears reveal the presence of Taenia eggs, products of a parasitic tapeworm infection. Which one of the following cells would be most effective in defence against this parasite? (A) Platelets (B) Erythrocytes (C) Neutrophils (D) Eosinophils (E) Monocytes

*The answer is A.* Scleroderma is characterized by vasculopathy and excessive collagen deposition in the skin and internal organs, such as the lung, gastrointestinal tract, heart, and kidney. The disease occurs four times as often in women as in men and mostly in persons aged 25 to 50 years. Progressive systemic sclerosis is characterized by widespread excessive collagen deposition. There is emerging evidence for the expansion of fibrogenic clones of fibroblasts. These clones display augmented procollagen synthesis, including increased circulating levels of type III collagen amino-propeptide. Tissue levels of the other proteins are not significantly altered in patients with scleroderma. Diagnosis: Scleroderma

A 45-year-old woman complains of severe headaches and difficulty swallowing. Over the past 6 months, she has noticed small, red lesions around her mouth as well as thickening of her skin. The patient has "stone facies" on physical examination. A skin biopsy in the patient described would most likely show a perivascular accumulation of which of the following extracellular matrix proteins? (A) Collagen (B) Elastin (C) Entactin (D) Fibronectin (E) Laminin

*The answer is D.* Scleroderma is an autoimmune disease of connective tissue. Circulating male fetal cells have been demonstrated in blood and blood vessel walls of many women with scleroderma who bore male children many years before the disease began. Accordingly, it has been suggested that scleroderma in these patients is similar to graft-versus-host disease. Antinuclear antibodies are common but are usually present in a lower titer than in patients with SLE. Antibodies virtually specific for scleroderma include (1) nucleolar autoantibodies (primarily against RNA polymerase); (2) antibodies to Scl-70, a nonhistone nuclear protein topoisomerase; and (3) anticentromere antibodies, which are associated with the "CREST" variant of the disease. The Scl-70 autoantibody is most common and specific for the diffuse form of scleroderma and is seen in 70% of patients. Autoantibodies to double-stranded DNA (choice B) are seen in patients with SLE. Autoantibodies to SS-A/SS-B (choice E) are seen in patients with Sjögren syndrome.

A 45-year-old woman complains of severe headaches and difficulty swallowing. Over the past 6 months, she has noticed small, red lesions around her mouth as well as thickening of her skin. The patient has "stone facies" on physical examination. Which of the following antigens is the most common and most specific target of autoantibody in patients with this disease? (A) C-ANCA (anti-proteinase-3) (B) Double-stranded DNA (C) P-ANCA (anti-myeloperoxidase) (D) Scl-70 (anti-topoisomerase I) (E) SS-A/SS-B

*The answer is E.* The case describes early rheumatoid arthritis. Early in this disease, patients present with bilateral, polyarticular joint pain, and tenderness. Joint deformity is generally a late-stage process indicative of severe joint pathology. Recognition of rheumatoid arthritis at an early stage is essential as pharmacologic intervention can delay the onset of joint deformity. The rheumatoid factor test is positive in about 70% of patients with rheumatoid arthritis; however, it may be negative early in the disease. The test detects the presence of autoantibodies (called rheumatoid factors), which bind to the Fc region of IgG. These autoantibodies are most commonly of the IgM isotype; however, IgG rheumatoid factors can also be found. While the presence of rheumatoid factor in a patient's serum is an aid to the diagnosis of rheumatoid arthritis, it is not used to make a definitive diagnosis. Rheumatoid factor can also be detected in patients with other types of connective tissue diseases as well as in normal individuals. The antinuclear antibody test is most commonly used as an aid to the diagnosis of systemic lupus erythematosus. This test may also be positive in some patients with rheumatoid arthritis, as well as several other connective tissue diseases. The Coombs test is used to make a diagnosis of immune hemolytic anemia. Anemia, common in patients with rheumatoid arthritis, is characterized as normocytic and normochromic and is not diagnosed with the Coombs test. The erythrocyte sedimentation rate is elevated in chronic inflammatory states and is not diagnostic for any particular disease. Radiographs would not be expected to be diagnostic during the early stage of the disease, prior to joint deformity.

A 45-year-old woman presents to her primary care physician with complaints of fatigue, malaise, and pain and stiffness in the shoulders, hands, and knees for the past 6 weeks. Joint stiffness was particularly pronounced in the morning and improved within an hour. Physical exam reveals tenderness and warmth of affected joints with no deformations. Laboratory tests reveal a normocytic and normochromic anemia, and analysis of synovial fluid from the knee revealed a white blood cell count of 10,000 with a predominance of neutrophils. Which test would be most useful in establishing a diagnosis of her condition? (A) Antinuclear antibody test (B) Coombs test (C) Erythrocyte sedimentation rate (D) Radiographs (E) Rheumatoid factor test

*The answer is A.* Sjögren syndrome (SS) is an autoimmune disorder characterized by keratoconjunctivitis sicca and xerostomia in the absence of other connective tissue disease. The production of autoantibodies, particularly antinuclear antibodies directed against DNA or nonhistone proteins, typically occurs in patients with SS. Autoantibodies to soluble nuclear nonhistone proteins, especially the antigens SS-A and SS-B, are found in half of patients with primary SS and are associated with more severe glandular and extraglandular manifestations. Autoantibodies to DNA or histones are rare. Organ-specific autoantibodies, such as those directed against salivary gland antigens, are distinctly uncommon. Autoantibodies to centromere proteins (choice A) are seen in the CREST variant of progressive systemic sclerosis. Diagnosis: Sjögren syndrome

A 45-year-old woman presents with a 1-year history of dry mouth and eyes. A biopsy of a minor salivary gland reveals infiltrates of lymphocytes forming focal germinal centers. Which of the following cellular organelles is a target for autoantibodies in this patient? (A) Centromere (B) Lysosome (C) Nucleus (D) Peroxisome (E) Plasma membrane

*The answer is E.* Superantigens are molecules of microbial origin which can non-specifically activate T cells. Superantigens bind to certain domains on MHC Class II molecules and to certain domains on the variable region of the beta chain on the T cell receptor, forming a bridge between the two cells. A superantigen can bind to T-cell receptors (TCRs) from a number of different T cells, as long as the TCRs share similar amino acid sequences in a key region of the variable portion of the beta chain. None of the other molecules, although expressed on T cells, binds directly with superantigens.

A 47-year-old woman developed toxic shock following an infection with a strain of Staphylococcus aureus that produced toxic shock syndrome toxin (TSST)-1. This toxin binds directly to MHC Class II molecules on macrophages and which molecule on T cells? (A) CD3 (B) CD40 ligand (C) Fas ligand (D) The gamma chain of the IL-2 receptor (E) The variable beta portion of the T-cell receptor

*The answer is E.* Immune complex (type III) hypersensitivity reactions cause vasculitis. Antigen-antibody complexes are either formed in the circulation and deposited in the tissues or formed in situ. Immune complexes induce a localized inflammatory response by fixing complement, which leads to the recruitment of neutrophils and monocytes. The vasculitis in patients with polyarteritis nodosa involves small to medium-sized muscular arteries. The diagnosis is usually made by biopsy of the skin, muscle, peripheral nerves, or the most affected internal organ (the kidney in this case). The most prominent morphologic feature of the affected artery is an area of fibrinoid necrosis (see photomicrograph). Other examples of type III hypersensitivity reactions include Henoch-Schönlein purpura (vascular IgA deposits) and vasculitis associated with hepatitis C infection. The other choices are uncommon mediators of vasculitis in patients with polyarteritis nodosa. Diagnosis: Polyarteritis nodosa

A 50-year-old man complains of fever, weight loss, abdominal pain, and bloody urine. Physical examination reveals red-purple discoloration of the skin. Serologic findings are inconclusive, but a positive P-ANCA test suggests an autoimmune disease. Biopsy of lesional skin discloses fibrinoid necrosis of a small muscular artery (shown in the image). Which of the following immune responses best explains the pathogenesis of inflammation and necrotizing vasculitis in this patient? (A) Antibody-dependent cellular cytotoxicity (B) Cytopathic autoantibodies (C) Delayed-type hypersensitivity (D) Immediate hypersensitivity (E) Immune complex disease

*The answer is E.* Hyperacute rejection occurs within minutes to hours after transplantation. It is manifested clinically as a sudden cessation of urine output, along with fever and pain in the area of the graft site. This immediate rejection is mediated by preformed antibodies and complement activation products. Lymphocytes and macrophages (choices A, B, and C) are associated with acute and chronic graft rejection. Diagnosis: Hyperacute graft rejection

A 52-year-old woman with a history of systemic hypertension and chronic renal failure undergoes kidney transplantation, but the graft fails to produce urine. A renal biopsy is diagnosed as "hyperacute transplant rejection." Graft rejection in this patient is caused primarily by which of the following mediators of immunity and inflammation? (A) Cytotoxic T lymphocytes (B) Helper T lymphocytes (C) Mononuclear phagocytes (D) Natural killer cells (E) Preformed antibodies

*The answer is B.* Graves disease is a type II hypersensitivity disorder caused by antibodies to the TSH receptor on follicular cells of the thyroid. Antibody binding to the TSH receptor stimulates a release of tetraiodothyronine (T4) and triiodothyronine (T3) from the thyroid into the circulation. Circulating T4 and T3 suppress TSH production in the pituitary. Sweating, weight loss, and tachycardia are evidence of the hypermetabolism typical of hyperthyroidism. Graves disease also causes exophthalmos. Delayed-type hypersensitivity (choice C) is seen in patients with poison ivy and graft rejection. Immune complex disease (choice E) is caused by deposition of immune complexes and complement activation. Diagnosis: Graves disease

A 53-year-old woman complains of progressive weight loss, nervousness, and sweating (patient shown in the image). Physical examination reveals tachycardia and exophthalmos. Her thyroid is diffusely enlarged and warm on palpation. Serum levels of thyroid-stimulating hormone (TSH) are low, and levels of thyroid hormones (T3 and T4) are markedly elevated. Which of the following mechanisms of disease best explains the pathogenesis of this patient's thyroid condition? (A) Antibody-dependent cellular cytotoxicity (B) Cytopathic autoantibodies (C) Delayed-type hypersensitivity (D) Immediate hypersensitivity (E) Immune complex disease

*The answer is B.* Type II hypersensitivity reactions are mediated by antibodies directed against fixed antigens. In this case, preformed antibodies in the patient's blood attached to foreign antigens (oligosaccharides) on the membranes of the transfused erythrocytes. At sufficient density, bound immunoglobulins fix complement. Once activated, the complement cascade leads to the destruction of the target cell through formation of a membrane attack complex. This type of complement-mediated cell lysis occurs in autoimmune hemolytic anemia. Antibody-dependent cell- mediated cytotoxicity (ADCC, choice A) involves cytolytic leukocytes that attack antibody-coated target cells. ADCC may be involved in the pathogenesis of some autoimmune diseases (e.g., autoimmune thyroiditis). Delayed-type hypersensitivity (choice C) occurs over a period of days and does not involve preformed antibodies. Diagnosis: Hemolytic anemia, jaundice

A 54-year-old woman is involved in an automobile accident and requires a blood transfusion. Five hours later, she becomes febrile and has severe back pain. Laboratory studies show evidence of intravascular hemolysis. It is discovered that type A Rh+ blood was given by mistake to this type B Rh+ patient. Which of the following best explains the development of intravascular hemolysis in this patient? (A) Antibody-dependent cellular cytotoxicity (B) Antibody-mediated complement fixation (C) Delayed-type hypersensitivity (D) Immune complex disease (E) Immediate hypersensitivity

*The answer is A.* Induction of immune suppression following renal transplantation usually involves a regimen of four drugs in the indicated categories, thus this choice could be used. The antilymphocyte anti-body should target all T cells, as both CD4 and CD8 cells play a crucial role in graft rejection. Thus, anti-CD3 antibodies are used since all T cells express this molecule as part of the antigen receptor complex. Only helper T cells express CD4 and only cytotoxic T cells express CD8, thus antibodies to either of these molecules would only lead to a decline in the respective population of cells. B cells express CD19 and CD20 and, although antibodies can play a role in graft rejection, their reduction would not prevent acute graft rejection. Calcineurin inhibitors include cyclosporine and tacrolimus. Both of these agents block the production of cytokines by effector T cells. The antiproliferative agents used to prevent transplant rejection are azathioprine, cyclophosphamide, and mycophenolate. Azathioprine is a purine analog that inhibits DNA replication. Mycophenolic acid inhibits de novo synthesis of purines. Compared to most other cells, lymphocytes are particularly sensitive to inhibition of purine syn-thesis as they are less able to generate this nucleotide by the purine salvage pathway. Cyclophosphamide is an alkylating agent that interferes with DNA replication. Sirolimus is a rapamycin that binds to the same intracellular protein as the calcineurin inhibitors (the FK506-binding protein)

A 56-year-old male with end stage renal disease receives a kidney transplant. He is treated post-transplant with an immune suppressive regimen including an antilymphocyte antibody, a calcineurin inhibitor, and an anti-proliferative drug in addition to corticosteroids. Which drugs, in addition to corticosteroids, is he given? (A) Anti-CD3, cyclosporine, and mycophenolate, respectively (B) Anti-CD4, cyclophosphamide, and tacrolimus, respectively (C) Anti-CD8, mycophenolate, and sirolimus, respectively (D) Anti-CD19, azathioprine, and mycophenolate, respectively (E) Anti-CD20, tacrolimus, and cyclosporine, respectively

*The answer is C.* RA is considered to be a T helper cell-driven autoimmune disease. Many of the pathological processes are mediated by T cell-derived cytokines which activate joint-associated cells for further cytokine secretion as well as proliferation and enhanced function. Under the influence of cytokines, the synovial membrane thickens to form a granulation tissue called pannus. Pannus consists of proliferating fibroblast-like synoviocytes, as well as infiltrating mononuclear cells. Cytokines and other mediators activate osteoclasts and chrondrocytes. Activated osteoclasts lead to bone resorption and deg-radation of bone. Activation of chrondrocytes leads to increased secretion of matrix metalloproteases which cause tissue degradation. Neovascularization occurs within pannus and is also stimulated by cytokines (chemokines). B-cell production of autoantibodies and immune complex deposition also occurs in RA and makes an important contribution to joint pathology; however, its role is not as great as that of cytokines. Complement activation is ongoing, particularly in synovial fluid where decreased complement levels indicate activation of complement; however, complement-mediated lysis of synovial membranes is not part of the pathologic process of RA. Mast cells may play a role in joint inflammation in RA; however, their role is not as important as that of cytokines. Cytotoxic T cells do not cause cartilage damage in RA.

A 57-year-old woman with chronic joint pain receives a diagnosis of rheumatoid arthritis (RA). What pathologic process, ongoing in the synovial membranes of her affected joints, is the most important contributor to joint damage? (A) B-cell autoantibody production and immune complex deposition (B) Complement-mediated lysis of synovial membranes (C) Cytokine activation of synoviocytes, chrondocytes, and osteoclasts (D) Cytotoxic T cell-mediated destruction of cartilage (E) Mast cell degranulation and infiltration of neutrophils

*The answer is B.* Control of intracelular pathogens is primarily dependent on intact cell-mediated immunity and control of extracellular pathogens is primarily dependent on intact humoral immunity. The pathogens causing disease in this child include both intracellular and extracellular organisms. Disseminated M. avium complex infections occur in patients with compromised cell-mediated immunity. Organisms causing otitis media requiring antibiotics are extracellular bacteria such as Streptococcus pneumoniae, Haemophilus inlfuenzae, or Moraxella catarrhalis. The agent that causes thrush and diaper rash, Candida albicans, is controlled by phagocytes. Uncontrolled growth of C. albicans occurs in patients with neutrophil dysfunctions or in patients whose T cells are unable to activate phagocytic macrophages.

A 6-month-old male is hospitalized with disseminated Mycobacterium avium complex. His history is significant for recurrent episodes of otitis media which responded poorly to antibiotics and episodes of severe thrush and diaper rash. Based on this history, an immunologic workup was done and the child was diagnosed with an immune deficiency disorder. Deficiencies of which aspect of the immune response could best account for the child's medical history? (A) B cells (B) B and T cells (C) Complement (D) Macrophages (E) Neutrophils

*The answer is B.* Corticosteroids are used in a variety of inflammatory diseases because they block the transcription of multiple genes involved in the inflammatory process. Inflammation is crucial to the pathogenesis of asthma, and is far more important than the early effects of mast cell degranulation lead-ing to initial bronchoconstriction. Inflammation of the airways occurs in the "late response" of allergic asthma, leading to an infiltration of eosinophils and mononuclear cells that promote continued asthma symptoms. The most effective therapies for asthma are aimed at reducing allergic inflammation, and affected genes include those encoding proinflamma-tory cytokines and chemokines as well as genes for adhesion molecules. In addition, corticosteroids block the metabolisms of arachidonic acid, decreasing the production of leukotrienes and prostaglandins that play important role in inflammatory diseases such as asthma. A reduction in inflammatory cells occurs in bronchial walls of patients successfully treated with inhaled corticosteroids. The other choices listed are not activities attributed to corticosteroids or their role in asthma therapy.

A 6-year-old boy is diagnosed with asthma after repeated episodes of wheezing and chest tightness. Frequent use of a bronchodilator does little to reduce the frequency of his symptoms. His visits to the emergency department decline after being placed on inhaled corticosteroids. What is the most important explanation for the clinical improvement seen with inhaled corticosteroids? (A) Decreased histamine production by mast cells (B) Decreased inflammatory cell infiltrates into bronchial wall (C) Decreased number of allergen-reactive B cells (D) Inhibition of Th17 cell function (E) Switch from Th2 cell activity to Th1 cell activity

*The answer is C.* Transplant rejection reactions have been traditionally categorized into hyperacute, acute, and chronic rejection based on the clinical tempo of the response and on the mechanisms involved. Acute rejection is characterized by an abrupt onset of azotemia and oliguria, which may be associated with fever and graft tenderness. A needle biopsy would be expected to show (1) interstitial infiltrates of lymphocytes and macrophages, (2) edema, (3) lymphocytic tubulitis, and (4) tubular necrosis. Neutrophilic vasculitis and fibrinoid necrosis (choice D) are seen in hyperacute rejection. Arterial intimal thickening (choice A), glomerulosclerosis (choice B), and tubular atrophy (choice E) are seen in chronic graft rejection. Diagnosis: Acute graft rejection

A 60-year-old woman with type 2 diabetes and end-stage renal disease receives a kidney transplant. Three weeks later, the patient presents with azotemia and oliguria. If this patient has developed acute renal failure, which of the following pathologic findings would be expected on renal biopsy? (A) Arterial intimal thickening and vascular stenosis (B) Glomerulosclerosis (C) Interstitial infiltrates of lymphocytes and macrophages (D) Neutrophilic vasculitis and fi brinoid necrosis (E) Tubular atrophy and interstitial fi brosis

*The answer is E.* Cachexia is a loss of body mass irreversible by nutrition. Tumor necrosis factor (TNF)-α is the main cytokine involved in cachexia. High levels of TNF-α in the blood lead to several metabolic changes in conjunction with increased energy utilization by the growing tumor. *TNF-α induces mobilization of fatty acids and protein catabolism leading to a breakdown of adipose tissue and skeletal muscle wasting.* The other cytokines listed are not responsible for causing cachexia.

A 66-year-old man with advanced pancreatic cancer develops cachexia. Which cytokine is primarily responsible for the cachexia seen in certain patients with cancer or debilitating infections? (A) Interferon-α (B) Interleukin-7 (C) Interleukin-17 (D) Transforming growth factor-β (E) Tumor necrosis factor-α

*The answer is B.* When the body experiences a significant inflammatory reaction, many physiological responses are generated. Interleukin-6, released into the circulation from activated macrophages and other cells, stimulates hepatocytes to release acute phase reactants. These proteins include components of the complement system, clotting system, and variety of others that play a role in promoting inflammation, or serving as opsonins. Some even down-regulate inflammation. Fibrinogen is a protein of the clotting system and causes an elevated ESR. In this test, a tube of heparinized blood is allowed to stand on end while the red blood cells settle to the bottom of the tube. When the red cells settle, they stack on top of one another. The more fibrinogen in the blood, the faster the red cells settle. Thus, the more inflammation, the more IL-6 is released and this, in turn, causes a greater release of acute phase reactants including fibrinogen. Another acute phase reactant is CRP, which functions as an opsonin. Both IL-1 and tumor necrosis factor (TNF)-α can also cause the release of acute phase reactants.

A 67-year-old woman is hospitalized with a fever of unknown origin. An elevated C-reactive protein (CRP) and erythrocyte sedimentation rate (ESR) suggest an ongoing systemic inflammatory response. Which cytokine is especially important in inducing elevations in CRP and ESR? (A) Interleukin-4 (B) Interleukin-6 (C) Interleukin-10 (D) Interleukin-12 (E) Interleukin-18

*The answer is C.* Repeated exposure to peanuts intensifies the immune reaction, indicating that the child most likely has developed an adaptive immune response to a peanut protein. A tolerogen would serve to diminish the immune response on repeated exposure. It is unlikely that an adjuvant is present that would intensify her allergic response to peanuts. Peanuts by themselves induced this response, but a hapten will not induce an immune response. The fact that the immune response intensifies on repeated exposu re effectively rules out an innate immune response.

A 7-year-old girl has a history of peanut allergy with symptoms that include generalized itching and hives after eating peanuts. Her symptoms became more severe with subsequent accidental exposures to peanuts. For this child, a peanut is most likely a(n) A. adjuvant. B. hapten. C. immunogen. D. innate immune system antigen. E. tolerogen.

*The answer is A.* LPS of gram-negative bacteria is recognized by LPS-binding protein in the bloodstream and tissue fluids. The LPS-LPS-binding protein complex is then delivered to the cell membrane of a macrophage, where resident LPS receptors, composed of a complex of proteins (TLR-CD 1 4-MD-2) bind the bacterial LPS. As a result of receptor engagement, the microbes are ingested and degraded, the macrophage is activated, and cytokine production and inflammation result. Actions of somatically generated receptors of B and T cells and of antibodies are part of the adaptive immune response as opposed to the innate response. Cytokines do not have antigen-specific activities, and killer activation receptors on NK cells recognize stress-related molecules on the surfaces of abnormal host cells.

A 76-year-old man is diagnosed with Escherichia coli septicemia. The initial immune response to E. coli (gram-negative bacteria) will include A. binding by LPS-binding proteins and delivery to receptors on macrophages. B. formation of specific somatically generated receptors to bind E. coli. C. generation and secretion of specific antibodies to recognize E. coli. D. production of E. coli-specific cytokines by lymphocytes. E. stimulation of killer activation receptors on NK cells.

*The answer is C.* Several leukocytes use perforin and granzyme to perform cell-mediated cytotoxicity, but only cytotoxic T cells express this activity from the given choices.

A CD31 cell that secretes perforin and granzyme is a (A) (γδ) T cell (B) Helper T cell (C) Cytotoxic T cell (D) Regulatory T cell (E) Natural killer T cell

*The answer is D.* Affinity maturation (choice A) refers to the process whereby immune receptors for specific antigen during a response are mutated to generate higher affinity, thereby allow-ing formation of memory B and T cells with the most reactive receptors to sense antigens upon re-encoun-ter. Antigen processing (choice B) is the process of breaking down and loading antigen onto MHC mol-ecules for surface recognition by responding T cells. Hematopoiesis (choice C) is the production of blood cellular components such as the many types of immune cells, and occurs in the bone marrow. Receptor editing (choice E) is a B cell process occurring in the bone marrow during development, and serves as a mode to recongure B-cell receptors that recognize self- antigen to identify foreign-antigens, otherwise, the self-reactive B cell will undergo apoptosis.

A blood sample from an individual with systemic lupus erythematosus was studied in a research project mapping T-cell receptor specificities. Many T cells were discovered to express receptors specific for autologous antigens. Failure of which process in the thymus leads to the large number of autoreactive T cells in the patient's blood? (A) Affinity maturation (B) Antigen processing (C) Hematopoiesis (D) Negative selection (E) Receptor editing

*The answer is D.* Only B cells express transmembrane immunoglobulins.

A cell captured by endocytosis using transmembrane immunoglobulin is a(n) (A) Immature myeloid-derived dendritic cell (B) Mature myeloid-derived dendritic cell (C) Follicular dendritic cell (D) B cell (E) Macrophage

*The answer is E.* Cell-mediated immunity is regulated by Th1 cells secreting INFγ when engaging macrophages that display intracellular non-self-antigen on MHC Class II molecules.

A cell producing cytotoxic compounds following Th1 cell activation is a(n) (A) Immature myeloid-derived dendritic cell (B) Mature myeloid-derived dendritic cell (C) Follicular dendritic cell (D) B cell (E) Macrophage

*The answer is B.* Both IgD and IgM are expressed on mature B cells when exiting the bone marrow. The IgD expression is lost when the B cell begins development in the germinal center.

A cell that expresses both IgM and IgD on the cell surface is a (A) B-1 cell (B) Naïve mature B cell (C) Centrocyte (D) Memory B cell (E) Plasma cell

*The answer is A.* Immature myeloid-derived dendritic cells are positioned in tissues to continuously sample extracellular debris for PAMPs and loading of intracellular MHC Class II.

A cell with a majority of MHC Class II located within intracellular compartments is a(n) (A) Immature myeloid-derived dendritic cell (B) Mature myeloid-derived dendritic cell (C) Follicular dendritic cell (D) B cell (E) Macrophage

*The answer is C.* IL-6 stimulates hepatocytes to secrete acute phase proteins.

A cytokine produced by macrophage to induce liver production of acute phase proteins is a (A) INF-γ (B) IL-4 (C) IL-6 (D) IL-10 (E) IL-17

*The answer is C.* MMPs are tissue degrading enzymes released from activated glial cells in the CNS as well as by infiltrating lymphocytes and monocyte-derived macrophages. These enzymes play an important role in the pathogenic process in MS and other immune-mediated diseases. Some studies have shown that β-interferon, a drug used in the treatment of MS, acts in part by down-regulating the production of MMPs. Metalloproteases are not involved in any of the other activities listed in the question, thus interfering with the function of MMPs would not lead to any of the other effects listed.

A drug in clinical trial for the treatment of multiple sclerosis inhibits matrix metalloprotease-9. If this drug leads to clinical improvement, how would it do so? (A) Causing a shift in cytokine production in the CNS (B) Degrading myelin antigens to prevent their presentation by oligodendrocytes (C) Helping protect the blood-brain barrier and the myelin sheath from degradation (D) Inducing apoptosis of activated CD8 cells (E) Inhibiting the adherence of leukocytes to the endothelium of the blood-brain barrier

*The answer is A.* This child has congenital thymic aplasia or DiGeorge syndrome, which is associated with a deletion in chromosome 22q11.2. Other syndromes associated with this deletion include velocardiofacial syndrome and conotruncal anomaly face syndrome. All these syndromes share characteristic facial features, cardiac abnormalities and recurrent infections. These syndromes represent a developmental defect involving the pharyngeal arches and pouches. Embryologic development of these areas, which include the thymus, parathyroids, heart, and face, is dependent on the migration of neural crest cells to the region of the pharyngeal pouches. Defects in RAG, adenosine deaminase, JAK3 molecules, which result in severe combined immunodeficiency and toll-like receptor signaling, are not associated with the distinctive physical features of this patient.

A male infant developed tonic convulsions a few days after birth and was found to be hypocalcemic. Of note, he was born with a cleft palate, widely spaced eyes, and ears residing lower than normal. Cardiac defects also became apparent in the neonatal period. A lateral view chest radiograph revealed a diminished thymic shadow. What genetic defect could account for the problems seen in this child? (A) A deletion in chromosome 22q11.2 (B) A mutation in adenosine deaminase (C) Defect in recombination-activating genes (RAG) (D) Janus tyrosine kinase 3 (JAK 3) deficiency (E) Toll-like receptor signaling defect

*The answer is C.* DiGeorge syndrome is a chromosomal defect that results in developmental anomalies of the branchial (pharyngeal) pouches and organs that develop from these embryonic structures (thymus, parathyroids, and aortic arch). These children present with tetany caused by hypoparathyroidism and deficiency of cellular immunity. They also have characteristic facial features ("angry look"). In the absence of a thymus, T-cell maturation is interrupted at the pre-T stage. DiGeorge syndrome has been corrected by transplanting thymic tissue. None of the other choices are associated with thymic aplasia. Diagnosis: DiGeorge syndrome

A neonate develops spastic contractions on the second postpartum day. Laboratory studies show hypocalcemia. MRI studies demonstrate aplasia of the thymus and parathyroid glands. What is the appropriate diagnosis? (A) Adenosine deaminase defi ciency (B) Common variable immunodefi ciency (C) DiGeorge syndrome (D) Transient hypogammaglobulinemia of infancy (E) Wiskott-Aldrich syndrome

*The answer is B.* The most important defense against extracellular bacteria are opsonins and neutrophils. Complement activation generates the opsonin C3b and attracts neutrophils to the area via the action of C5a. IgM is the most efficient class of antibody at activating complement. IgM can function in neutralization of toxins; however, that is not the most effective protective function (choice D). IgM is not itself an opsonin (choice E), in contrast to IgG. No antibody class can induce bacterial cell lysis directly (i.e., without the assistance of complement) (choice C). IgM does not participate in antibody-dependent cell cytotoxicity (choice A), a function which is carried out against viral-infected cells, but not against extracellular bacteria. Lastly, natural killer cells are not important in defense against extracellular bacteria.

A person developed an extracellular bacterial infection, and IgM was made in response. What is the most important protective function of IgM in this infection? (A) Antibody-dependent cell cytotoxicity (B) Complement activation (C) Direct lysis of bacterial cells (D) Neutralization of bacterial toxins (E) Opsonization

*The answer is D.* CD4+ and CD8+ T cells can only respond to antigens once they have been processed and loaded onto major histocompatibility complexes (MHC) and displayed by antigen presenting cells. B cells recognize antigens directly. Upon initial B-cell encounter with antigen, B cells traffick to the paracortex of the lymph nodes to interact with helper T cells. This interaction involves CD40 ligand/CD40 which promote B-cell expansion as plasma cells (choice A). Once receiving the CD40 signal, the B cells traffick back into the lymphoid follicle and form a germinal center. Antigen is presented to the developing B cells by follicular dendritic cells, FDC (choice B). Only B cells expressing the highest affinity immunoglobulin receptors will remain bound to the FDC. All other B cells will undergo apoptosis. Helper T cells then produce the appropriate cytokines for isotype switching and cell survival signals are provided through CD40 ligand/CD40 interactions. Some of the B cells differentiate into antibody-producing plasma cells while others become memory B cells which leave the lymph node and traffick to the bone marrow where they remain as long-lived plasma cells. Receptor editing occurs in the bone marrow when B cells are undergoing central tolerance and involves rearrangement of Ig genes (choice C). Like all adaptive receptors each one is specific for only one antigen unlike innate receptors which recognize conserved sequences on pathogens (choice E).

A person develops a viral infection and both T and B cells become activated to fight the infection. In which way is antigen recognition by B cells different from antigen recognition by T cells? (A) B cells home to the paracortex of lymph nodes where they recognize the antigens trapped by helper T cells (B) B cells recognize the antigens that have been processed and presented by follicular dendritic cells (C) B cells undergo receptor editing to change receptors that fail to bind to an antigen (D) B cells utilize membrane immunoglobulin molecules to bind to antigen in its natural state (E) The antigen receptors on a single B cell have a broad specificity, and are able to recognize several chemically unrelated antigens

*The answer is E.* The first antibody isotype produced by B cells is IgM, that is, it is the first type of B cell antigen receptor. IgA (choice A) is the isotype of antibody synthesized and secreted primarily in the mucosa. IgD (choice B) is an isotype of antibody found in the blood in low levels and its exact function is unknown. IgE (choice C) is an isotype produced to mediate antibody-dependent cell cytotoxicity of parasites. It binds to Fc receptors on eosinophils, basophils, and mast cells and upon antigen binding results in granule release. IgG (choice D) is the highest concentration of antibody in the serum, mediates complement activation, opsonization, and can cross the placenta to protect fetus.

A person is experiencing a primary infection with the virus. B cells activated in a primary infection secrete which class of antibody first? (A) IgA (B) IgD (C) IgE (D) IgG (E) IgM

*The answer is C.* Anergy is a state of non reactivity that occurs when a lymphocyte receives a stimulus through its TCR or BCR in the absence of the additional appropriate signals provides by antigen-presenting cells or T cells. Allergy involves the degranulation of mast cells following binding of antigen to lgE molecules already affixed to the mast cell surfaces. Apoptosis is the programmed death of a cell through degradation of its nucleic acids. Autoimmunity is the active response of the immune system against self epitopes. Hypersensitivity is a response mediated by activated lymphocytes or their products. Allergy is one form of hypersensitivity.

A state of T-lymphocyte nonresponsive ness that occurs following peptide + major histocompatibility complex (pMHC) engagement is known as A. allergy. B. apoptosis. C. anergy. D. autoimmunity. E. hypersensitivity.

*The answer is A.* The antibody secreted in mucosal tissues is IgA, all other antibody isotypes, IgD (choice B), IgE (choice C), IgG (choice D), and IgM (choice E) are found predominantly in other fluids.

A viral infection began in the respiratory tract. Which antibody class would best protect respiratory epithelial cells from viral infection? (A) IgA (B) IgD (C) IgE (D) IgG (E) IgM

*The answer is D.* The predominant antibody isotype within the blood (site of viremia) is IgG. It functions to activate complement cascade, antibody-dependent cell cytotoxicity and opsonization. IgM (choice E) is also of high levels within the blood but not as significant as IgG. IgA (choice A) is found pre-dominantly in the mucosal compartments. IgD (choice B) is found in low levels in the blood and its function is unknown. IgE (choice C) does not contribute to viral infection control

A virus spreads from the respiratory tract and causes viremia. Which antibody class would be most important in fighting the virus as it spreads through the body? (A) IgA (B) IgD (C) IgE (D) IgG (E) IgM

*The answer is A.* Upon antigen uptake, macrophages through toll-like receptor signaling, produce interleukin-12 which in turn promotes the differentiation of Th1 cells from naïve CD4+ T cell pools. These Th1 cells can then produce IFN-γ which is bound by macrophage IFN-γ receptors signaling to promote macrophage phagocytosis. GM-CSF (choice B) serves to differentiate granulo-cytes from stem cells. IL-1 (choice C) is produced by macrophages upon activation and stimulates inammation. MCP (choice D) is a chemotaxis fac-tor for both macrophages and T cells and at least at this point does not play a role in macrophage activation. TGF-β (choice E) is produced by many cells and serves to stimulate embryogenesis but functions to suppress activities of T cells.

Activation of macrophages is best achieved by which cytokine? (A) Interferon gamma (IFN-γ ) (B) Granulocyte monocyte colony-stimulating factor (GM-CSF) (C) Interleukin-1 (D) Macrophage chemotactic protein (MCP) (E) Transforming growth factor beta (TGF-β)

*The answer is A.* Activation of complement has three outcomes: promotion of inflammation through recruitment of phagocytes (C3a, C5a), opsonization to enhance phagocytosis by complement receptor-expressing phagocytes (C3b, C4b), and target cell killing (C5b-C9). Expression of Toll-like receptors is constitutive on phagocytes although this expression can be increased (choice B). Neutralization (choice C) is the process of blocking pathogen binding to host receptors (usually accomplished by antibodies), and is not a function of complement activation. Complement activation does not directly result in T-cell proliferation (choice D). Complement activation does not directly promote antibody binding to Fc receptors (choice E).

Activation of the complement system, directly results in which one of the following outcomes? (A) Enhanced phagocytosis (B) Expression of Toll-like receptors on phagocyte cell surface (C) Enhancement of immune-mediated neutralization (D) Proliferation of T cells (E) Interaction of Fc receptors with antibodies bound to antigens on the pathogen cell surface

*The answer is E.* The congenital disorder Bruton X-linked agammaglobulinemia appears in male infants at 5 to 8 months of age, the period during which maternal antibody levels begin to decline. The infant suffers from recurrent pyogenic infections and severe hypogammaglobulinemia. There is an absence of both mature B cells in peripheral blood and plasma cells in lymphoid tissues. The genetic defect, located on the long arm of the X chromosome, is an inactivating mutation of the gene for B-cell tyrosine kinase, an enzyme critical to B-lymphocyte maturation. Wiskott-Aldrich syndrome (choice D) is also an X-linked genetic disease but is characterized by defects in both B-cell and T-cell functions (i.e., humeral and cellular immunity). DiGeorge syndrome (choice A) is a developmental disorder characterized by thymic and parathyroid aplasia. Diagnosis: X-linked agammaglobulinemia of Bruton

An 8-month-old boy with a history of recurrent pneumonia is found to have almost no circulating IgG. Cellular immunity is normal. His brother had this same disease and died of echovirus encephalitis. His parents and sisters have normal serum levels of IgG. What is the appropriate diagnosis? (A) DiGeorge syndrome (B) Isolated IgA deficiency (C) Severe combined immunodeficiency (D) Wiskott-Aldrich syndrome (E) X-linked agammaglobulinemia of Bruton

*The answer is B.* X-linked agammaglobulinemia (XLA), also called Bruton agammaglobulinemia, is a con-genital immune deficiency disease characterized by a lack of B-cell production by the bone marrow. B cells express CD19 and CD20, and these two markers are exploited in flow cytometry studies to enumerate B cells in the peripheral blood. The lack of these cells in peripheral blood of a male infant is diagnostic for XLA. Natural killer cells express CD16+ and CD54+ and are not affected in this disease. Absent serum IgG, very low levels of IgM is inconsistent with XLA since with-out B cells no IgM could be produced. Infants with this disease have maternal IgG in their serum which declines at about 5 to 6 months of age, leaving them at increased risk for infections such as those seen in this patient. Infants with XLA have normal to elevated levels of CD3+ T cells. Lymphopenia is not seen, as the majority of circulating lymphocytes are T cells.

An 8-month-old male has recurrent episodes of seri-ous infections including otitis media, chronic sinusitis, and bacterial pneumonia. He is diagnosed with X-linked agammaglobulinemia after an immunologic workup. Which test result is consistent with this diagnosis? (A) Absent CD16+ and CD54+ cells in peripheral blood (B) Absent CD19+ and CD20+ cells in peripheral blood (C) Absent serum IgG, very low levels of IgM (D) Decreased CD3+ cells in peripheral blood (E) Lymphopenia

*The answer is E.* Antibodies bind to epitopes on antigens to identify them or tag them for destruction by other elements of the immune system. They are synthesized only by B cells and plasma cells. An antibody molecule contains two ( lgD, lgG, lgE, and serum lgA), four (secretory lgA) , or ten (secreted lgM ) identical epitope-binding sites. An antibody monomer contains two identical light chains and two identical heavy chains. Self-recognition is not required for antibody molecules.

Antibodies (immunoglobulins) A. are synthesized and secreted by both B and T cells. B. bind to several different epitopes simultaneously. C. contain four different light chain polypeptides. D. recognize specific epitopes together with self molecules. E . tag antigens for destruction and removal.

*The answer is B.* Binding to an epitope causes a conformation change in the Fc portion of the antibody molecule. Fc receptors (FcRs) recognize and bind to the conformationally altered antibody molecule, and this engagement of epitope-bound antibody by FcRs stimulates phagocytosis of cells and molecules '1agged" by antibodies for destruction. Complement receptors bind and facilitate the phagocytosis of cells and molecules tagged by complement components or fragments. Killer activation receptors, pattern recognition receptors, and toll-like receptors do not recognize antigen-antibody complexes.

Antibody-mediated recruitment of macrophages occurs through action of A. complement receptors. B. Fe receptors. C. killer activation receptors. D. pattern recognition receptors. E. toll-like receptors.

*The answer is E.* Innate receptors or pattern recognition receptors (PRRs) recognize conserved amino acid and carbohydrate sequences on the surface of pathogens. This detection allows a rapid response in the form of phagocytosis and cytokine production to lead to adaptive immune activation. Antigen receptors on B and T cells are the product of gene segment rearrangement in order to devise receptors which can recognize various antigens from differing pathogens. Affinity maturation (choice A) is the process whereby activated B cells compete for binding with follicular dendritic cells in the germinal centers in order to generate the B cells synthesizing the highest affinity antibodies. T cells require interaction with MHC-peptide complexes in order to undergo activation, whereas B cells can recognize antigens in their natural state (choice B). The constant regions are not identical in nature, especially since soluble T-cell receptors do not bind to Fc receptors while B-cell receptors if solubilized will interact with Fc receptors (choice C). Specificity of both B- and T-cell antigen receptors is determined during development within the bone marrow and thymus, respectively, with those cells recognizing self-antigens undergoing apoptosis (choice D).

Antigen receptors on T and B cells share which similar feature? (A) Affinity maturation occurs following antigen recognition for both receptor types (B) Interaction with MHC molecules is required for antigen recognition by both receptor types (C) The constant regions of both receptor types are identical (D) The specificity of both receptor types is determined following exposure of mature cells to antigen (E) The variable portions of both receptor types are generated by random recombination of genes

*The answer is C.* APCs express TLR, FcR, and MHC. TLR (choice B) are innate pattern recognition receptors (PRR) which detect microbial antigens and initiate proinflammatory cytokine production by phagocytes (APCs as well). FcR (choice E) are important in the process of opsonization (FcR binding to antibody bound to the antigen on the pathogen surface and subsequent phagocytosis via this interaction). All nucleated somatic cells express MHC class I while only the professional APCs (B cells, macrophages, and dendritic cells) also express MHC class II. MHC displays antigens for recognition by T cell receptors (TCRs) (choice A).

Antigen-presenting cells (APCs) are required for T-cell recognition of specific antigen and activation. APCs accomplish this task by presenting antigen in the context of which of the following molecules? (A) T-cell receptor (TCR) (B) Toll-like receptor (TLR) (C) Major histocompatibility complex (MHC) (D) Killer inhibitory receptor (KIR) (E) Fc receptor (FcR)

*The answer is C.* Cells bear many types of receptors, each capable of specifically binding a different ligand. The signals generated by the binding of various combinations of receptors on the surface of a given cell are integrated by that cell and used to determine the action to be taken.

Cells of the immune system are triggered by the binding of surface receptors. In general, the action taken is determined by A. a single receptor per cell . B. a single type of receptor found on all cells. C. the integration of signals generated by multiple receptors on single cells. D. multiple receptors that bind soluble ligands only. E. nonspecific receptors capable of binding a wide array of ligands.

*The answer is C.* Dansyl is a hapten in that it meets three criteria: It is a synthetic molecule; by itself, it does not stimulate an immune response; and when it is coupled to an immunogenic molecule, an immune response is stimulated toward both dansyl and the immunogen. An adjuvant increases the intensity of an immune response. A carrier molecule is also an immunogen. An immunogen is a substance that stimulates an immune response. A tolerogen causes unresponsiveness.

Dansyl (5-dimethylaminonaphthalene-1 -sulfonyl) is a synthetic molecule that binds to receptors on certain B cells but does not stimulate them to produce dansyl-specific antibodies unless it is first conjugated to a larger, immunogenic molecule such as bovine serum albumin. These findings indicate that dansyl is a(n) A. adjuvant. B. carrier. C. hapten. D. immunogen. E. tolerogen.

*The answer is A.* Professional antigen presenting cells are those that express both MHC class I and II molecules. Basophils (choice B), eosinophils (choice C), mast cells (choice D), and neutrophils (choice D) all do not express MHC class II and thus cannot present antigens to CD4 T cells. In addition, these cell types do not express the machinery for processing of exogenous antigens.

Dendritic cells, macrophages, and what other cell types are considered "professional antigen presenting cells," capable of antigen presentation to T helper cells? (A) B cells (B) Basophils (C) Eosinophils (D) Mast cells (E) Neutrophils

*The answer is D.* Acute transplantation rejection is mediated by T cells and represents an alloreactive response against MHC antigens expressed by the donated tissues. Prior to transplantation, MHC molecules of the host are matched with MHC molecules of a potential donor. Rejection reactions are made less likely the more MHC molecules the two have in common. However, finding a perfect match, except between identical twins, is highly unlikely. This is because of the tremendous heterogeneity of MHC molecule expression in the human population. Even if a single MHC molecule is mismatched between the donor and the recipient, immunologic reactivity of the recipient will occur and could lead to an acute rejection reaction. Multiple T-cell clones are activated in response to a single unre-lated MHC molecule because each allogeneic MHC molecule offers several distinct epitopes for T cells of the recipient to react against. Thus, the polyclonal nature of the immune reaction against transplanted tis-sues makes the response far stronger than the immune response against pathogens. Other forms of rejection of transplanted tissue can occur. Chronic rejection, occurring about 6 months posttransplantation, is characterized by a delayed type hypersensitivity reac-tion of host T cells against donor endothelial cells leading to atherosclerotic vascular damage in a grafted organ. Other nonimmunologic factors, including the use of nephrotoxic immune suppressants and reactivation of cytomegalovirus (though not necessarily the host's immune response to the virus) can also play a role in chronic rejection. The presence in the recipient of preformed antibodies to the donor ABO blood group antigens would lead to hyperacute rejection beginning within hours of transplantation. This type of rejection can be avoided by using donors and recipi-ents with the same blood type. While donor dendritic cells are present in the grafted tissue and can interact with recipient T cells in all activation, they do not play a role in acute rejection through presentation of the recipient own proteins.

Despite the immunosuppressive regimen given to the patient in the above question, within a month posttransplant, signs of rejection began to appear as evidenced by rising creatinine levels. Acute rejection was diagnosed by biopsy. What is the immunopathologic basis for the rejection of this transplanted kidney? (A) Delayed-type hypersensitivity reaction of host T cells leading to atherosclerotic vascular damage in the grafted kidney (B) Dendritic cells in the transplanted kidney presenting recipient antigens to and activating recipient cytotoxic T cells (C) Presence in the recipient of preformed antibodies to the donor ABO blood group antigens (D) Polyclonal activation of T cells that cross-react with allogeneic MHC molecules on the transplanted kidney (E) T-cell response to reactivated cytomegalovirus causing damage to renal tubular epithelial cells

*The answer is E.* CD4 (choice A) and CD8 (choice B) are involved in MHC-TCR interactions. CD45 (choice C) is a cell surface marker for hematopoietic cells. CD28 (choice D) indeed binds to CD80/86 but the signal conveyed is stimulatory rather than inhibitory as for CD152 or CTLA-4.

Downregulation of T-cell activation is achieved by the binding of which molecule on the T cell with CD80/86 on the dendritic cell? (A) CD4 (B) CD8 (C) CD45 (D) CD28 (E) CD152

*The answer is D.* Delayed-type hypersensitivity is defined as a tissue reaction involving lymphocytes and mononuclear phagocytes, which occurs in response to a soluble protein antigen and reaches greatest intensity 24 to 48 hours after initiation. In the initial phase, foreign protein antigens or chemical ligands interact with accessory cells bearing class II HLA molecules. Protein antigens are actively processed into short peptides within phagolysosomes and are presented on the cell surface in conjunction with the class II HLA molecules. The latter are recognized by CD4+ T cells (choice A), which become activated to synthesize an array of cytokines. The cytokines recruit and activate lymphocytes, monocytes, fibroblasts, and other inflammatory cells. Suppressor T cells are CD8+ (choice B). Class I HLA molecules (choice C) provide targets for cell-mediated cytotoxicity. GlyCAM-1 (choice E) is a cell adhesion molecule involved in lymphocyte trafficking. Diagnosis: Delayed-type hypersensitivity

During the physical examination of a 22-year-old man, a purified protein derivative isolated from Mycobacterium tuberculosis is injected into the skin. Three days later, the injection site appears raised and indurated. Which of the following glycoproteins was directly involved in antigen presentation during the initiation phase of delayed hypersensitivity in this patient? (A) CD4 (B) CD8 (C) Class I HLA molecules (D) Class II HLA molecules (E) GlyCAM-1

*The answer is C.* Following an initial B-cell proliferative event, cells termed plasmablasts differentiate into long-lived plasma cells which traffic to the bone marrow and reside there, actively producing antibodies to the specific antigen against which they were originally expanded (choice D). Circulatory memory B cells (choice A) also exist but do not actively secrete antibody and appear to serve as replacements for the long-lived plasma cells in the bone marrow over time (choice B). TLR signaling in memory B cells has not been described to impact antibody production as these circulating cells appear not to synthesize the antibodies (choice E).

Following an initial expansion of B cells in response to microbial peptides, a memory pool of B cells is generated and maintained in the individual in many cases throughout life. Recently, it was discovered that two memory cell types were generated in response to microbial challenge. Upon rechallenge with the microbe, the patient is protected from infection. Which one of the following explanations accounts for this observation? (A) Circulating memory B cells are actively producing antibodies in the absence of antigen (B) Circulating memory B cells will produce antibodies, however, only upon encounter with the microbial antigen (C) Long-lived plasma cells are actively producing antibodies in the absence of antigen (D) Long-lived plasma cells will produce antibodies, however, only upon encounter with the microbial antigen (E) TLR signaling to the circulating memory B cells will induce rapid production of antibodies in response to antigen encounter

*The answer is A.* Perforin forms pores in the target cells and allows the passage of granzymes into the cytoplasm of the cells. Granzyme then initiates the process of apoptosis. Only cytotoxic T cells and NK cells produce perforin/granzyme while dendritic cells (choice B), eosinophils/basophils (choice C), macrophages/ neutrophils (choice D), and mast cells (choice E) do not produce these molecules.

If a person had a genetic defect affecting perforin production, which cells and immune function would be affected? (A) Cytotoxic T cells and natural killer cells/cell killing (B) Dendritic cells/antigen presentation (C) Eosinophils and basophils/granule production (D) Macrophages and neutrophils/phagocytosis (E) Mast cells/fusion of granules to cell membrane

*The answer is B.* Identification of self tells the immune system that the cell or molecule recognized is not a foe. Natural killer cells use this mechanism of self-recognition to halt their attack on cells that they perceive to be abnormal. Receptor generation by B and T cells occurs independently of initial encounter with self molecules. Pattern recognition receptors, on the other hand, are genetically programmed to recognize nonself. By triggering an attack on a cell expressing the self molecule, an immune recognition molecule violates its "nonaggression pacf' with the cells and molecules of the host and establishes an internal coup known as autoimmunity.

Immune recognition of molecules belonging to self is important to A. activate natural killer cells of the innate immune system . B. determine the safety of interacting with the molecule. C. induce somatic generation of a B- or T-lymphocyte receptor for the molecule. D. stimulate binding by pattern recognition receptors. E. trigger an attack on the cell expressing the self molecule

*The answer is C.* lsotype switching is a process in which rearranged VDJ genes within a memory B cell become juxtaposed through DNA excision from an upstream (5') C region gene with a different C region gene farther downstream (3') . Affinity maturation of antibody for its epitope is independent of isotype. For B cells that have "selected" their maternal or paternal immunoglobulin variable region genes, there are no "do-overs:' Both junctional diversity and somatic hypermutation involve the antigen-binding site for immunoglobulin and do not appear to influence a switch from one isotype to another.

In a patient who later developed an allergy to a certain antigen , the initial response to the antigen consisted of immunoglobulin of the lgM class. However, over time, antigen-specific lgE came to be predominant. This change from an lgM to an lgE response is caused by A. affinity maturation. B. allelic exclusion. C. isotype switching. D. junctional diversity. E. somatic hypermutation.

*The answer is B.* Interferon-γ promotes isotype switching from IgM to IgG, IL-4 (IL-13) induces switching to IgE; IL-2 and IL-7 do not influence isotype switching (choices A, C, D, and E). T helper cells and B cells interact through CD40 ligand-CD40 interactions whereas CTLA-4 and B7 are costimulatory signals, inhibitory and stimulatory, respectively. CD20 and antigen are not T helper cell-derived factors and cannot promote isotype switching.

In order for class switching from IgM to IgG to occur, B cells must receive two signals, one generated following binding to T helper cells and the other secreted by helper T cells. What are these two T helper cell-derived molecules? (A) Antigen and IL-2 (B) CD40 ligand and interferon-γ (C) CTLA-4 and IL-4 (D) B7 and IL-7 (E) CD20 and IL-13

*The answer is E.* All of these activities can follow activation of phagocytic cells by the recognition and binding of lipopolysaccharide via their toll-like receptors. Activated phagocytes can secrete various cytokines that can be involved in chemotaxis and activation of other leukocytes. Among these cytokinesis IL-12, which stimulates natural killer cells to increase their production of IFN-γ, which, in turn, promotes the differentiation of CD4+ ThO cells into Th 1 cells.

In the presence of microbe-derived lipopolysaccharide, A. antigen-presenting cells may secrete IL- 1 2 . B. release of cytokines results in leukocyte activation. C. stimulation of IFN-γ secretion activates leukocytes. D. Th0 cells further differentiate into Th1 cells. E. all of the above

*The answer is C.* The molecules on the virus that are not on host cells are the pathogen-associated molecular patterns. The pattern recognition receptors are found on host cells and molecules. MHC I and II molecules are present on all nucleated host cells but not on viruses. The somatically generated receptors are on host T and B lymphocytes.

Influenza viruses infect humans and elicit an immune response that is often insufficient to protect the individual from sickness or death. Which of the following structures are on influenza viruses, allowing them to be recognized by the human immune system? A. MHC I molecules B. MHC II molecules C. Pathogen-associated molecular patterns D. Pattern recognition receptor E. Somatically generated receptors

*The answer is B.* Lymphocytes including bone marrow-derived (B cells) , thymus-derived (T cells), and natural killer (NK) cells derive from lymphoid lineage cells. They account for fewer than 40% of blood leukocytes; neutrophils are the most numerous. Lymphoid lineage cells are agranular leukocytes and are also poorly phagocytic.

Lymphoid lineage cells A. are the most numerous leukocyte population. B. consist of B, T, and N K cells. C. contain conspicuous cytoplasmic granules. D. differentiate from myeloid cell precursors. E . phagocytize debris and foreign cells.

*The answer is E.* Microbes express conserved amino acid and carbohydrate sequences on their cell (or viral capsid) surfaces called *pathogen-associated molecular patterns (PAMPs)*. These sequences are recognized by pattern recognition receptors (PRRs) on the surface of phagocytes. Antigen receptors (choice A) are adaptive immune response receptors structured to recognize specific antigens unique to pathogens. Complement receptors (choice B) are expressed on phagocytes and function to enhance phagocytosis of complement-coated (C3b, C4b) pathogens. Fc receptors (choice C) are expressed on numerous cell types and function to bind to immunoglobulin for opsonization and antibody-dependent cell cytotoxicity (NK cells and granulocytes such as mast cells). Membrane immunoglobulin (choice D) is expressed only on B cells and serves as the B cell antigen receptor

Macrophages recognize microorganisms through the interaction of microbial substances with what type of receptors on macrophages? (A) Antigen receptors (B) Complement receptors (C) Fc receptors (D) Membrane immunoglobulin (E) Pattern recognition receptors

*The answer is C.* NKT cells do express TCRs generated (like those of other T cells) by DNA rearrangement and junctional diversity. They are either CD4+ or CD4+CD8+. Despite this, their TCRs recognize lipid-related molecular fragments presented by the nonclassical class I molecule CD1d. They do not synthesize or express immunoglobulins.

NKT cells A. are usually CD8 single positive cells. B. bind epitopes presented by MHC class II molecules. C. express TCRs generated by DNA rearrangement and junctional diversity. D. recognize carbohydrates and complex proteins. E. synthesize immunoglobulin and display it on their cell surfaces.

*The answer is C.* Natural killer (NK) cells scrutinize nucleated cells using killer activation receptors (KARs) that detect stress molecules (MICA and MICB) expressed on cells in response to intracellular infection. Epstein-Barr virus infection causes cells to display stress molecules and at the same time, decrease their expression of MHC class I molecules. Engagement of KAR triggers the lytic activity by the NK cells. Killing of the target cells will proceed unless killer inhibition receptors (recognizing MHC class I molecules on the targets cells) are appropriately engaged. If KIR are not engaged at a sufficient level, the KAR-initiated lysis proceeds. Complement receptors bind activated fragments of complement that occur in the extracellular environment. Fc receptors bind antibodies that engage extracellular environment antigens. NK cells do not express immunoglobulins.

Natural killer (NK) cells lyse Epstein-Barr virus-infected B cells with deficient MHC I expression. The NK receptors that initiate the lytic activity are A. complement receptors. B. Fc receptors. C. killer activation receptors. D. killer inhibition receptors. E. monomeric immunoglobulin receptors.

*The answer is B.* Bacterial mannose and lipopolysaccharide (LPS) (choice A) serve as ligands for pattern recognition receptors found on phagocytes. Histamine is a vasoactive amine released upon degranulation of mast cells, while C3b is an opsonin (choice C). IL-7 and IL-16 function in cell activation and survival but do not operate in chemotaxis (choice D). While leukotriene B4 is a pro-inflammatory product of mast cell activation, G-CSF is a growth stimulating factor for cells (choice E). Only the cleavage product C5a and IL-8 are both recruiting factors for neutrophils

Neutrophils are attracted to the sites of extracellular bacterial infections by which two important chemo-tactic substances? (A) Bacterial mannose and lipopolysaccharide (B) Complement C5a and interleukin-8 (CXCL-8) (C) Histamine and complement C3b (D) Interleukin-7 and interleukin-16 (E) Leukotriene B4 and granulocyte colony-stimulating factor (G-CSF)

*The answer is C.* Pattern recognition receptors of the innate immune system bind structural patterns composed of proteins, sugars, and lipids that are found on microbes but are not found in the human host. This mechanism allows for a rapid and precise recognition of potential pathogens. In contrast, B and T lymphocytes are components of the adaptive immune system in which somatically generated receptors recognize precise molecular details of antigens as opposed to broad structural characteristics found in pathogen-associated molecular patterns.

Pathogen-associated molecular patterns (PAM Ps) A. allow B and T lymphocytes to recognize bacteria and destroy them. B. are cysteine-rich peptides that form channels in bacterial membranes. C. are recognized by pattern recognition receptors of the innate immune system. D. closely resemble host cell surface proteins and sugars. E . induce secretion of interferons by vi rally infected host cells.

*The answer is B.* IL-1 and TNF (choice A) are cytokines produced by activated phagocytes and T cells promoting inflammation. IL-10 and TGF-β (choice C) are cytokines produced by Th2, macrophages, and regulatory T cells to suppress the immune response as a mechanism of regulating immunity. In addition, IL-10 can promote Th2 differentiation. TGF-β (combined with IL-6) can promote differentiation of Th17 cells involved in autoimmune diseases. TGF-β alone can promote differentiation of Th0 into regulatory T cells. IL-12 and IFN-γ (choice D) are cytokines which promote Th1 differentiation and activation of macrophages (enhance phagocytosis). Interferon alpha and beta (choice E) are cytokines induced by viral infections which result in the establishment of the antiviral state. Since IgE bound to Fc receptor is critical for antibody-dependent cell cytotoxicity of helminths, the cytokines of importance are those which promote isotype switching from IgM to IgE, and production and chemotaxis of eosinophils that is, IL-4 and IL-5 respectively.

Persons with helminth infections mount immunologic responses that involve IgE and eosinophils. Which two cytokines are most important for these responses to occur? (A) IL-1 and tumor necrosis factor (TNF) (B) IL-4 and IL-5 (C) IL-10 and transforming growth factor beta (TGF-β) (D) IL-12 and interferon gamma (IFN-γ) (E) IFN-α and IFN-β

*The answer is C.* IgA is the isotype associated with mucosal immunity. While it can be produced in other anatomical regions, it is abundant in the mucosa. The bone marrow (choice A) contains long-lived plasma cells producing anti-bodies. Germinal centers (choice B) within the cervical nodes could also produce IgA, but B cells in the lamina propria produce significantly more IgA. The thoracic duct (choice D) is the lymph vessel responsible for returning cells and fluid from the lymph back into circulation. The white pulp of the spleen (choice E) contains areas of B cell differentiation into plasma cells and can produce IgA, but as IgA is an impor-tant neutralizing agent in the mucosa, it is localized in high levels within the mucosa.

Plasma cells secreting IgA are especially abundant in which body site? (A) Bone marrow (B) Germinal centers of cervical lymph nodes (C) Lamina propria of mucosa (D) Thoracic duct (E) White pulp of the spleen

*The answer is E.* Positive selection refers to recognition of MHC class I (by CD8) or MHC class II (by CD4) by double-positive (CD4+ CD8+) thymocytes. Single positive thymocytes (and T cells) are either CD4 + or CD8+ and recognize either MHC class II (CD4) or MHC class I (CDS), but not both. Cortical thymocytes acquire a nascent TCR as well as CD4 and CD8 surface molecules, resulting in formation of double-positive (CD4+CD8+) thymocytes. Precursor T cells migrate or traffic from the bone marrow to the thymus before acquiring CD4 and CD8, which they will do as cortical thymocytes. Cells that fail to complete positive selection undergo programmed cell death (apoptosis) .

Positive selection refers to A. the ability of single positive cells to bind both MHC class I and II. B. cortical thymocytes' acquisition of TCR. C. migration of stem cells to the thymus to become T cells. D. programmed cell death of single positive T cells. E . recognition of MHC by CD4+CD8+ thymocytes.

*The answer is A.* A given B cell or plasma cell expresses a single maternal or paternal allele of a chromosome pair. This process, known as allelic exclusion, applies to both heavy and light chain genes. An additional exclusion allows for the expression of only a κ (chromosome 2) or λ. (chromosome 22) gene, never both within the same cell. Allelic exclusion has only a slight impact on genetic variation. lsotype switching, junctional diversity, and random V(D)J joining occur after allelic exclusion.

Serum immunoglobulins containing both maternally and paternally derived Vκ light chains are found within an individual. A given B cell , however, expresses only maternally derived or paternally derived Vκ chains but never both. This finding is the result of A. allelic exclusion. B. antibody diversity. C. isotype switching. D. junctional diversity. E. random VD and VDJ joining.

*The answer is A.* T-cell receptors are randomly generated prior to any engagement with antigens. Phagocytic cells use phagocytosis and pinocytosis to internalize antigens without regard to the specificity of the ingested material. T cells do not synthesize immunoglobulins. The selection for receptors recognizing a widely expressed set of microbial molecules is a property of toll-like receptors, not of T-cell receptors. The genomically encoded pattern recognition receptors are toll-like receptors.

T cells recognize epitopes they have never before encountered by A. randomly generating enormous numbers of TCRs prior to antigenic encounter. B. sampling the environment using phagocytosis and pinocytosis. C. synthesizing immunoglobulins specific for a wide variety of epitopes. D. selecting widely expressed molecules as TCR ligands. E. using genomically encoded pattern recognition receptors.

*The answer is B.* Activation (choice A), differentiation (choice D), and proliferation (choice E) are T-cell outcomes following TCR interaction with peptideMHC complexes in the presence of cos-timulatory signals (CD28-CD80/86 interactions). Apoptosis (choice C) is a signal generated in T cells through FasFasL (CD95-CD95L) and TNFTNFR (tumor necrosis factorTNF receptor) interactions. Anergy is a state of unresponsiveness, that is, upon re-exposure to the same antigen, even in the presence of appropriate costimulation, the T cells remain unable to proliferate although they do seem to be able to pro-duce cytokines.

T cells stimulated by peptide-MHC complexes, diplayed on antigen presenting cells, in the absence of costimulation undergo which one of the following processes? (A) Activation (B) Anergy (C) Apoptosis (D) Differentiation (E) Proliferation

*The answer is D.* CD4 (choice A) and CD8 (choice B) are molecules which interact with antigen presenting cell MHC molecules to stabilize the MHC-peptide-TCR interaction. CD45 (choice C) is a cell surface protein found on hematopoietic cells. CD152 (choice E) or CTLA-4, cytotoxic T lymphocyte antigen-4, is expressed on activated T cells (CD4 and CD8) and interacts with B7 (CD80/86) molecules to suppress further T-cell activities (form of peripheral regulation). Only B7, expressed by antigen presenting cells, are used to engage CD28 to promote T-cell activation. CTLA-4 has a higher afnity for B7 than does CD28, thus providing a means of regulating T-cell activation status.

T helper cells interacting with antigen-presenting dendritic cells require signals generated by the molecular interactions of the T-cell receptor with the MHC-peptide complex. Additionally, costimulation is required to amplify the initial TCR signals provided through the T cell CD28 molecule interaction with which one of the following dendritic cell molecule(s)? (A) CD4 (B) CD8 (C) CD45 (D) CD80/86 (E) CD152

*The answer is C.* CD8+ T cells are restricted to the recognition of pMHC I complexes. CD3 molecules are associated with the TCR on the T-cell surface and are found on both mature CD4+ and CD8+ T cells. CD4+ T cells are restricted to the recognition of pMHC class II complexes. MHC class III molecules include complement components C4, Bf, and C2 and are not involved in T-cell recognition.

T-cell receptors, when coexpressed with CD8 molecules, are restricted to recognizing and binding peptide fragments associated with A. CD3 molecules. B. CD4 molecules. C. MHC class I molecules. D. MHC class II molecules. E. MHC class III molecules.

*The answer is A.* The photograph shows a woman with ophthalmopathy secondary to the hyperthyroidism of Graves disease. Graves disease is due to the formation of antibodies to the thyroid stimulating hormone receptor. These antibodies constantly stimulate the thyroid to secrete thyroid hormones in the absence of normal physiological feedback mechanisms that control hormone secretion. None of the other conditions listed are associated with hyperthyroidism. Goodpasture syndrome is an autoimmune disease affecting the lung and kigney. Hashimoto thyroiditis is mediated by T cell infiltration into the thyroid, resulting in hypothyroidism. Myasthenia gravis is an immune-mediated condition resulting in profound muscle weakness. Sjogren syndrome is an inflammatory connective tissue disease leading to dry eyes, dry mouth, and enlarged parotid glands.

The 56-year-old woman in the accompanying photograph has been under the care of an endocrinologist and receiving radioactive iodine for a hyperthyroid condition in which thyroid-stimulating immunoglobulins are present in the serum. What is the diagnosis? (A) Graves disease (B) Goodpasture syndrome (C) Hashimoto thyroiditis (D) Myasthenia gravis (E) Sjogren syndrome

*The answer is A.* Mannose-containing residues of glycoproteins on certain microbes activate the mannan-binding lectin pathway of complement. Killer activation receptors on NK cells recognize stress-related molecules on the surfaces of abnormal host cells. Antigen-antibody complexes are not required to initiate the alternative complement pathway. Toll-like receptor binding to pathogen-associated molecular patterns stimulates synthesis and secretion of the cytokines to promote inflammation and recruitment of leukocytes to the site of infection.

The alternative complement pathway is initiated by A. cell-surface constituents that are recognized as foreign to the host. B. mannose-containing residues of glycoproteins on certain microbes. C. stimulation of killer activation receptors on NK cells. D. the formation of antibody-antigen complexes. E. toll-like receptor binding to pathogen-associated molecular patterns.

*The answer is A.* The T-cell receptor (TCR) is a heterodimer composed of αβ or γδ polypeptide chains. Neither the αβ or γδ heterodimers nor their associated molecules (CD3 and CD247) are linked by disulfide bonds. TCR recognize pMHC complexes on antigen-presenting cells. TCRs are found only on the surfaces of T cells and are not soluble.

The basic structure of a T cell receptor consists of A. a membrane-bound αβ or γδ heterodimer. B. a complex of disulfide-linked heavy and light chains. C. covalently linked CD3 and CD247 molecules. D. peptide-MHC complexes. E. soluble antigen-binding homodimers.

*The answer is C.* Neutralization is the blocking by antibody of structures on microbes and toxins that allow them to bind to host cell surfaces. Agglutination is the aggregation or clumping of cells or particles bound together by antibodies (usually lgM or dimeric lgA) . Complement activation is initiated by the attachment of the C1 component of complement to epitope bound antibody (lgM or lgG). Opsonization is the increased phagocytic uptake of cells or molecules tagged by antibodies (usually lgG1) or membrane-bound C3b or C4b. The precipitin reaction results from the assembly of large antigen-antibody complexes that precipitate from solution.

The binding of antibodies to microbial epitopes or soluble molecules in a manner that inhibits the ability of these microbes/molecules to bind to host cell surfaces is termed A. agglutination. B. complement activation. C. neutralization . D. opsonization. E. precipitin reaction.

*The answer is A.* The classical pathway of complement begins with the recognition of antigen-antibody complexes by the first component of complement, C1q. Subsequent steps in the classical pathway involve activation of components C4, C2, C3, and the production of C3 convertase leading to the production of C5 convertase and entry into the membrane attack complex. Antigen binding by lgA does not activate the classical pathway.

The classical pathway of complement begins with A. activation of C1 . B. cleavage and activation of C4, C2, and C3. C. lgA binding to a specific epitope. D. initiation of membrane attack complex formation. E . production of C3 convertase.

*The answer is C.* The heavy chain constant regions determine immunoglobulin isotypes: mu (μ, lgM), delta (δ, lgD), gamma (γ , lgG), epsilon (ε, lgE), and alpha (α, lgA). Fab fragments are enzymatic cleavage products of immunoglobulin monomers. (lmmunoreceptor) tyrosine activation motifs are not present on immunoglobulin molecules. Variable domains show extensive amino acid sequence variability among immunoglobulins, even within the same isotype.

The constant regions of the five major types of heavy chains of immunoglobulin molecules dictate the molecule's A. epitope. B. Fab fragment. C. isotype. D. tyrosine activation motif. E. variable domain .

*The answer is C.* Maturation indicates that the dendritic cell (DC) has been stimulated by Toll-like receptor binding to pathogen components. This, in turn, upregulates expression of B7 (costimulatory molecules) on the surface of the DC. Together with the MHC-peptide complex, the DC can now provide sufficient signaling to promote T-cell activation. Such interactions deficient in B7 result in anergy (choice A). Apoptosis (choice B) signals are the result of ligand-eceptor interactions involving Fas-Fas ligand or TNF-related interactions. Ignorance (choice D) is the result of antigens being sequestered from access to T cells; thus, the T cells remain ignorant of the antigen presence (immune-privileged sites such as the sex organs utilize this mechanism). Suppression (choice E) is an active process mediated by cytokines or regulatory cells such as regulatory T cells.

The difference between tolerance and immunity depends upon the maturation status of the antigen presenting dendritic cells. What is the T-cell outcome of an antigen presentation event by a mature dendritic cell? (A) Anergy (B) Apoptosis (C) Activation (D) Ignorance (E) Suppression

*The answer is A.* TdT adds or removes nucleotides when the ends of V, (D), and/or J gene segments are exposed. This process, known as junctional diversity, occurs during DNA rearrangement. This process occurs in addition to the fusion of VDJ segments of the heavy chain and occurs prior to a B cell's exposure to antigen. The light chain constant region (CL) never joins with constant region (CH) domains of the heavy chain to make a polypeptide. A crossover between maternal and paternal VL alleles is an exceedingly rare event, and TdT is not involved.

The role of terminal deoxynucleotidyl transferase (TdT) in development of antibody diversity is to A. add/remove nucleotides of V, D, and J genes. B. fuse VD and J segments together in heavy chains. C. increase binding affinity of antibody for antigen . D. join CL to CH1, CH2, CH3, or CH4 domains. E. transfer VL alleles from maternal to paternal chromosomes.

*The answer is C.* Antibodies generated by the vaccination are produced by antigen-specific memory B cells which reside in the bone marrow. These antibodies (most likely IgG) can promote opsonization (choice A), Fc receptor-mediated phago-cytosis, and complement activation (choice D) as well as ADCC (choice E). However, the infectious agent in most cases will have colonized tissues by the time these mechanisms act to limit growth. The most critical function of a vaccine-mediated humoral response is to generate antibodies which block attachment of infectious agents to host cell surfaces or neutralization. This blockade completely inhibits microbial colonization and ensures protection from the pathogen. Extravasation (choice B) refers to the process whereby immune cells can leave the circulation and enter tis-sues generally to migrate into areas of inflammation in order to destroy pathogens invading host tissues.

Vaccination operates to generate a humoral immune response to the immunogen(s). Which one of the fol-lowing represents the critical function of the resultant humoral response in protecting vaccinated patients from future infections by targeted pathogenic agents? (A) Opsonization (B) Extravasation (C) Neutralization (D) Complement activation (E) Antibody-dependent cell cytoxicity (ADCC)

*The answer is E.* The case describes a child with T-cell dysfunction, as demonstrated by his infection with Pneumocystis jeroveci a fungal organism thought to be part of the normal oropharyngeal fl ora and that causes disease in patients with compromised T-cell-mediated immunity. As the T cells are normal in number, their activation must be impaired, and his current infection supports this idea. The fact that the patient has elevated IgM and very low levels of IgG and IgA, suggests that the T cells cannot interact adequately with B cells to induce class switching. In addition to cytokine stimuli, the interaction of CD40 on the B cell with CD40 ligand (also known as CD154) on the T cell is essential to stimulate class switching in B cells. This molecule is also essential for contact-mediated activation of macrophages by Th1 cells, and activated macrophages are important for the control of P. jeroveci. The other immunologic processes listed are not affected in this disease.

What immunologic function is impaired in the child and best explains the infection in the above case? (A) Antigen presentation by dendritic cells (B) B-cell migration to follicles in secondary lymphoid tissue (C) Differentiation of B and T cells from precursor cells (D) Expression of functional antigen receptors on T cells (E) T-cell activation of macrophages and class switching in B cells

*The answer is B.* Myasthenia gravis results from antibody directed against the acetylcholine receptor. Receptors bound by antibody are not stimulated by acetylcholine, and the neurotransmitter is then degraded by acetylcholine esterase. Use of the affected muscles results in progressive weakness as more and more acetylcholine is degraded. Improvement of muscle weakness with rest is attributed to regeneration of acetylcholine stores. Acetylcholine esterase inhibitors are used in therapy of this disease. Abdominal pain, weight loss, and bloody diarrhea have a number of causes including inflammation of colonic mucosa in inflammatory bowel disease. Ascending weakness and paresthesia suggest Guillian Barre syndrome, which is due to an immunologic attack on the peripheral nerve myelin sheath. Fatigue, pallor, and mild jaundice suggest a form of anemia such as autoimmune hemolytic anemia mediated by antired blood cell antibodies. Goiter, palpitations, and proptosis of the eyes are consistent with Graves disease caused by antibody-mediated stimulation of the thyroid gland.

What is the immunopathogenesis of myasthenia gravis? (A) Antibody-mediated hemolysis of red blood cells (B) Antibody-mediated interference with neuromuscular transmission (C) Antibody-mediated stimulation of the thyroid gland (D) Immunologic attack of peripheral nerve myelin sheath (E) Inflammation of colonic mucosa

*The answer is D.* Estimates of the frequency of this disease range from 1 in 700 to 1 in 3,000 live births. The other diseases listed have a much lower incidence. The frequency of Bruton agammaglobulinemia has been estimated to be 1 in 250,000 to 1 in 379,000 live births. Common variable immunodeficiency in contrast to its name is not common, occurring in about 1 in 10,000 to 1 in 50,000. Leukocyte adhesion deficiency is extremely rare, hav-ing been reported in only a few hundred infants in the United States. X-linked severe combined immunodeficiency is one of several diseases under the umbrella term of severe combined immunodeficiency diseases. The frequency of this group of diseases is estimated to be between 1 in 50,000 and 75,000 live births.

What is the most common primary immune deficiency disorder? (A) Bruton agammaglobulinemia (B) Common variable immunodeficiency (C) Leukocyte adhesion deficiency (D) Selective IgA dficiency (E) X-linked severe combined immunodeficiency

*The answer is E.* B-cell differentiation in the bone marrow begins with the first committed cell in B-cell development, the pro-B cell. This cell expresses the B-cell marker, CD19. Rearrangement of the variable genes of the heavy chain occurs at this stage, and the cell is considered a pre-B-cell when it expresses the first version of its antigen receptor, called the pre-B cell receptor. Further development of the B cell from this stage relies on signals generated by the receptor and delivered to the nucleus by a signal transduction molecule known as Bruton tyrosine kinase. Bruton agammaglobulinemia or X-linked agammaglobulinemia (XLA) is associated with a defect in the gene for btk. The BTK gene is required for the proliferation and differentiation of B lymphocytes. Therefore, mutations in BTK results in deficient B-cell development and subsequent lack of antibody production in response to antigens.

What is the pathogenesis of X-linked agammaglobulinemia? (A) An inability of B cells to differentiate into plasma cells (B) An inability of B cells to undergo cognate binding with T helper cells (C) An inability of B cells to undergo VDJ gene recombination (D) An inability of plasma cells to secrete IgM (E) An inability of pro-B cells to differentiate into B cells

*The answer is E.* IgA is a structural dimer and is held together by the J chain (choice D). IgM can form pentamers also held together by the J chain. B cells produce IgA and the secretory piece permits translocation across mucosal epithelia, subsequent secretion and protection from proteolysis. The hinge region (choice C) is the flexible portion connecting the Fab (choice A) and Fc regions of an antibody. Fc receptors (FcR) (choice B) are receptors on cells which bind Fc regions of antibodies. These interactions result in cell signaling, activation, opsonization, and/or cytokine production.

What structural feature is uniquely found on IgA in breast milk and not found on serum IgM? (A) Fab (B) FcR (C) Hinge region (D) J chain (E) Secretory piece

*The answer is E.* Enzymatic cleavage of the immunoglobulin monomer by pepsin occurs distal to the variable domain and distal to heavy-heavy chain disulfide bonds, which remain intact, resulting in a molecule with two epitope-binding sites. Interchain disulfide bonds are unaffected by pepsin cleavage. The epitope-binding site remains intact on pepsin cleavage of the heavy chain . Papain cleavage of the immunoglobulin monomer occurs distal to the variable domain but proximal to the heavy-heavy chain disulfide bond, resulting in two separate epitope-binding Fab fragments. Pepsin enzymatically degrades the CH2 portion of the immunoglobulin molecule resulting in fragments that rarely, if ever, form crystals.

When an immunoglobulin molecule is subjected to cleavage by pepsin, the product(s) A. are individual heavy and light chains. B. can no longer bind to antigen. C. consist of two separated antigen-binding fragments. D. crystallize during storage in the cold. E . is a dimeric antigen-binding molecule.

*The answer is E.* B cells (choice A) are associated with responses of type I, II, and III hypersensitivity. Cytotoxic T cells (choice B) can produce a type IV hypersensitive (cell-mediated) response by killing target cells expressing the antigen such as in rheumatoid arthritis. Eosinophils (choice C) can participate in type I as they express Fc receptors for IgE. Natural killer cells (choice D) are recruited participants in inflammatory responses but are not primarily responsible for hypersensitivity reactions. Delayed-type hypersensitivity reactions are primarily driven by Th1 cells producing IFN-γ.

Which cell type is primarily responsible for the inflammation seen in poison ivy rash? (A) B cells (B) Cytotoxic T cells (C) Eosinophils (D) Natural killer cells (E) Th1 cells

*The answer is C.* All of the cells listed have the capacity to kill target cells. Cytotoxic T cells (choice A) or activated CD8+ T cells kill primarily through Fas-FasL interactions or production of granzyme/perforin. Eosinophils, basophils, mast cells, macrophages, and neutrophils kill targets by secreting reactive oxygen species and enzymes. NK (choice D) and NKT (choice B) cells use killer activating receptor (recognize MHC class I-negative cells) signaling and produce granzyme/ perforin. Th1 (choice E) cells can produce TNF-α and IFN-γ which has been shown to cause cell death.

Which cells utilize reactive oxygen and nitrogen species and lysosomal enzymes to kill pathogens? (A) Cytotoxic T cells (B) Natural killer T (NKT) cells (C) Macrophages (D) Natural killer (NK) cells (E) Th1 cells

*The answer is C.* This drug is an antimalarial used in the initial therapy of SLE. Its mechanism of action is primarily aimed at neutrophil and monocyte/macrophage chemotaxis and phagocytosis. It has been shown to inhibit the release of lysosomal contents by stabilizing the membranes of these organelles. In addition, it blocks arachidonic acid metabolism thereby preventing the release of prostaglandins. Azathioprine, cyclophosphamide, and methotrexate are immune suppressant, cytotoxic drugs, and methylprednisolone is a corticosteroid. All of these drugs increase the risk of infection and neoplasia in patients.

Which drug reduces inflammation without causing generalized immune suppression. What drug is she given? (A) Azathioprine (B) Cyclophosphamide (C) Hydroxychloroquine (D) Methotrexate (E) Methylprednisolone

*The answer is E.* Granulomas are the product of the immune system effort to control Mycobacterium replication. The two cells responsible for formation of the granuloma are macrophages and Th1 cells. There are relatively few cytotoxic T cells (choice A), dendritic cells (choice B), and natural killer cells (choice D) present in or near the granuloma. Eosinophils (choice C) do not appear to participate in the immune response against Mycobacterium.

Which immune system cell is primarily responsible for the formation of granuloma in the lungs of tuberculosis patients? (A) Cytotoxic T cells (B) Dendritic cells (C) Eosinophils (D) Natural killer cells (E) Th1 cells

*The answer is D.* Cytotoxic T cells (choice A) are CD8+ T cells which recognize antigens presented on MHC class I molecules, and act to kill targets expressing those antigens. Dendritic cells (choice B) are specialized antigen presenting cells. Macrophages (choice C) and neutrophils (choice E) are phagocytes. Only natural killer cells recognize MHC class I expression on the surface of cells and kill those cells that have lost expression.

Which immune system cells recognize body cells with reduced expression of MHC class I molecules? (A) Cytotoxic T cells (B) Dendritic cells (C) Macrophages (D) Natural killer cells (E) Neutrophils

*The answer is A.* Bloodborne basophils and tissue resident mast cells are responsible for allergic responses caused by the release of vasoactive amines within their cytoplasmic granules. Dendritic cells, lymphocytes, and monocytes all play roles in adaptive immune responses but are not the actual effector cells in allergic reactions. Neutrophils actively destroy invasive bacteria.

Which of the following cells are important effector cells in allergic reactions? A. Basophils B. Dendritic cells C. Lymphocytes D. Monocytes E. Neutrophils

*The answer is C.* CD4+CD25+ Treg cells inhibit various responses against self-epitopes as well as some responses against epitopes associated with infectious agents and tumors. Antigen-presenting cells do not have this capacity. Anergized cells are inactive. Follicular dendritic cells are involved in the display of antigen to B cells and T cells in the lymph node follicles. Naive T cells require activation before they can begin to carry out any of their effector functions.

Which of the following cells have been implicated in the prevention of autoimmune responses (e.g . , inflammatory bowel disease) and in the prevention of some nonself responses? A. Antigen-presenting cells B. Anergized T cells C. CD4+CD25+ Treg cells D. Follicular dendritic cells E. Naive T cells

*The answer is D.* Dendritic cells are the usual participants in the activation of naive cells. Anergized T cells remain refractory to subsequent engagement of pMHC and remain quiescent. B cells do not require binding of pMHC for activation. Mast cells become activated and degranulated via the binding of antigen to IgE molecules already affixed to the mast cell surfaces. Natural killer cells do not have receptors for binding pMHC.

Which of the following cells require interaction with both pMHC and a set of costimulatory second signals from an antigen-presenting cell (usually a dendritic cell) to become activated? A. Anergized T cells B. B cells C. Mast cells D. Naive T cells E. Natural killer cells

*The answer B.* Dendritic cells use two mechanisms to sample their extracellular environment. One, phagocytosis, internalizes by endocytosis molecules and cells that are bound to the cell's su rface receptors. The other, macrophagocytosis, involves the engulfment of extracellular fluids by cytoplasmic projections. Basophils, eosinophils, macrophages, and neutrophils do not use macrophagocytosis.

Which of the following cells sample their extracellular environment by macropinocytosis? A. Basophils B. Dendritic cells C. Eosinophils D. Macrophages E. Neutrophils

*The answer is A.* β-defensins can attach to microbes and make them more susceptible to ingestion by phagocytes. Peptidoglycan of bacterial cell walls is degraded by lysozyme, not by fatty acids or DNase. Lysozyme does not act on RNA and DNA. The lacrimal fluid contains lysozyme that acts on microbial peptidoglycan, not on host phagocytes.

Which of the following is a correct pairing of a soluble molecule with its microcidal action in the respiratory tract? A. β-defensins increase microbial susceptibility to phagocytosis. B. DNase enzymatically damages microbial membranes. C. Fatty acids of commensal microbes degrade microbial peptidoglycan. D. Lacrimal secretions facilitate ingestion of microbes by phagocytes. E. Lysozyme degrades DNA and RNA produced by pathogenic microbes.

*The answer is A.* The bone marrow is a primary lymphoid organ. Lymph nodes, Peyer's patches, spleen, and tonsils are all secondary lymphoid organs.

Which of the following is a primary lymphoid organ? A. Bone marrow B. Lymph node C. Peyer's patch D. Spleen E. Tonsil

*The answer is C.* Normal stomach pH is between 1 .0 and 3.0. The values given for the respiratory tract, skin, upper gastrointestinal tract, and vagina are abnormal.

Which of the following is an example of a normal physiologic pH barrier to microbial colonization? A. Respiratory tract pH between 9.0 and 1 1 .0 B. Skin pH of approximately 8.0 C. Stomach pH between 1 .0 and 3.0 D. Upper gastrointestinal tract pH between 6.5 and 7.5 E . Vaginal pH of approximately 7.0

*The answer is E.* Because the mucosal immune system is constantly exposed to so many nonself-epitopes that are essentially harmless, it is tolerant to most of them. Although it can respond to microbes that pose a pathogenic threat, the mucosal system generally avoids the development of chronic inflammation because of the damage that could be inflicted on the delicate mucosal linings. The immunologic environment is generally described as more Th2-like than Th1-like. lgG is present at far lower levels than is lgA.

Which of the following is characteristic of the mucosal immune system? A. A vigorous response is made to all nonself-antigens encountered. B. Chronic inflammation makes an inhospitable environment for microbes. C. IL-2 and IFN-γ contribute to a Th1 -like environment. D. Secretion of lgG predominates over secretion of lgA. E. Tolerance to foreign antigens is the norm rather than the exception.

*The answer is B.* Bacterial toxins are often very immunogenic. An individual should normally not make adaptive immune responses against her or his own plasma proteins. A 500-Da plasma protein from a chimpanzee is small enough to "fall under the radar" of the adaptive immune system, most likely because it lacks sufficient numbers of epitopes. A cholesterol molecule is most likely not immunogenic irrespective of size. Immune responses in normal individuals will not be directed against carbohydrates that their tissues or fluids express.

Which of the following is most likely to induce the greatest adaptive immune response in a 25-year-old man? A. 250,000-Da plasma protei n from the same 25-year-old h u man male B. 1 50,000-Da toxin produced by bacteria C. 500-Da plasma protein from a chimpanzee D. 400-Da cholesterol molecule from an unrelated human female E . 200-Da carbohydrate molecule common to all species

*The answer is A.* Most of the antibody generated in the human MALT (mucosa-associated lymphoid tissues) is of the lgA isotype. lgE, lgG, and lgM are present, but at far lower levels, and lgD is essentially absent.

Which of the following is the predominant immunoglobulin isotype secreted in the human MALT ? A. lgA B. lgD C. lgE D. lgG E. lgM

*The answer is C.* Of those cell types listed, only dendritic cells can process peptide fragments and load them on MHC II molecules for presentation. Lymphocytes, whether of the CD4+, CD8+, or -y8 type, cannot do this. Neutrophils can ingest peptides and degrade them but do not synthesize MHC II molecules.

Which of the following naive cells load peptide fragments into MHC class II molecules? A. CD4+ T cells B. CDS+ T cells C. dendritic cells D. γδ T cells E. neutrophils

*The answer is B.* Eosinophils contain cytoplasmic granules that serve as potent agents against infection by parasitic worms (helminths). Basophils do not migrate to the site of an infection. Although lymphocytes, monocytes, and neutrophils will migrate to infection sites, eosinophils uniquely migrate to sites of parasitic worm infection.

Which of the following types of cells are notable for their presence at the sites of helminth infections? A. Basophils B. Eosinophils C. Lymphocytes D. Monocytes E. Neutrophils

*The answer is A.* The attenuated vaccine, in which the organism is still capable of some degree of infection and reproduction, is likely to produce a stronger immune response than are the other types of vaccines, in which the virus is incapable of doing so. In general, the safer the vaccine (in terms of risk of reversion to a virulent wild type), the less effective it is (in terms of offering protection).

Which of the following types of vaccines would most likely evoke the best and most long-lasting protective immune response against rubeola (measles)? A. Attenuated vaccine B. DNA vaccine C. Extract vaccine D. Killed vaccine E. Recombinant vaccine

*The answer is A.* In the formation of granulomas, two cell types predominate, Th1 cells and macrophages. Activated macrophages produce IL-12 which promotes Th1 differentiation. In turn, Th1 cells produce IFN-γ to further activate macrophages. IL-2 (choice B) is important in promoting early proliferation of Th1 cells but chronic infection (granuloma) with Mycobacterium would result in activation-induced cell cytotoxicity (AICC) driven by IL-2. IL-5 (choice C) and IL-10 (choice D) is produced by Th2 cells which do not play a role in Mycobacterium immunity. TNF-α (choice E) is produced by Th1 cells but in vivo studies suggests it only synergizes with IFN-γ in the containment of the bacterium.

Which one of the following cytokines plays the most important role in protection against intracellular growth (reactivation) of Mycobacterium tuberculosis? (A) Interferon-γ (B) Interleukin-2 (C) Interleukin-5 (D) Interleukin-10 (E) Tumor necrosis factor

*The answer is A.* In fact, about 90% of all T cells that are produced are eliminated (apoptosis) during negative selection. Thymic medullary epithelial cells express the Aire gene (autoimmune regulator gene) and synthesize all self-antigens for presentation on MHC molecules to maturing thymocytes. Mature dendritic cells (DCs) also participate in the presentation of self-antigens to developing single positive T cells. Those T cells that respond too strongly or weakly undergo apoptosis (undetermined signals) while those responding intermediately are provided survival factors and complete maturation, and they migrate out of the thymus to seed the lymphoid organs in the body. While there may be some expansion of nonself-reactive T cells (choice B) due to provided survival/growth factors, this is not the major role of negative selection. DCs do mature but only serve to present self-antigens (choice C). TCR expression is completed following positive selection (choice D). Differentiation of CD4+ T cells (choice E) occurs after antigen exposure in the periphery (tissue, spleen, or lymph node).

Which one of the following represents the major role of negative selection in the thymus? (A) Elimination of self-reactive T cells (B) Expansion of nonself-reactive T cells (C) Maturation of professional antigen presenting cells such as dendritic cells (D) Expression of T-cell receptors on mature T cells (E) Differentiation of Th1 and Th2 CD4+ T cells

*The answer is E.* Defects in the complement pathway are associated with IC diseases due to the fact that complement is important for the removal of ICs that occur during normal B-cell immune responses to pathogens. ICs are created by soluble (or cell surface) antigen interaction with secreted antibodies. This normally results in opsonization and phagocytosis; however, another mechanism of clearance is mediated in the spleen and liver by macrophages. C3 cleavage product, C3b, binds to the ICs and is attached to complement receptors on red blood cells. These cells then pass through the spleen and liver, and a tissue factor removes the ICs from the red blood cells and resident macrophages take up the ICs for degradation (choices A-D).

Which one of the following represents the mechanism by which immune complexes (ICs) are normally cleared from the circulation? (A) ICs are solubilized by C3a (B) ICs are solubilized by C5a (C) Factor I releases complement-bound ICs to bind with complement receptors found on splenic/ hepatic neutrophils (D) Red blood cells capture ICs from blood via Fc receptors (E) C3b solubilizes ICs and attaches to red blood cells via complement receptors

*The answer is D.* Type I (immediate hypersensitivity) (choice A) is an immune response mediated by IgE triggering of mast cell and basophil degranulation. Type II (choice B) are antibody-mediated responses in which antibodies against cellular or extracellular matrix antigens may deposit in any tissue expressing the target antigen. Thus, the disease is normally tissue-specific. An example of type II hypersensitivity is Goodpasture syndrome and myasthenia gravis. Type III (choice C) are immune-complex mediated events where antibodies to soluble antigens bind and form complexes which deposit in areas of high pressure such as blood vessel branches and the kidney glomeruli. An example of type III hypersensitivity is systemic lupus erythematosus. Poison ivy induces a delayed-type hypersensitivity in which CD4+ T cells are activated and an inflammatory response develops.

Which type of hypersensitivity is associated with reactions to poison ivy oil? (A) Type I (B) Type II (C) Type III (D) Type IV


Kaugnay na mga set ng pag-aaral

CCENT 1.2 Compare and Contrast TCP and UDP Protocols

View Set

economics concepts and choices chapter 17

View Set

FPM 3250 Week 8 Reading Notes - Elbow, Radioulnar Joint, and Wrist

View Set

The Thirteen Colonies and The British Empire (1607-1754) Terms & Definitions

View Set

FDMA 2855-Social Media Marketing Tools

View Set

Cambridge English Profile Level C1

View Set